Practice Question Bank

Ace your homework & exams now with Quizwiz!

The double-blind, randomized, parallel-group, placebo-controlled GetGoal-L trial compared the effects of lixisenatide (n=328) with placebo (n=167) in patients with type 2 diabetes mellitus inadequately controlled using basal insulin. A secondary outcome of the GetGoal-L trial was the proportion of patients who achieved the A1c goal (less than 7.0%) at 24 weeks. Which of the following types of data are the primary outcome of the GetGoal-L trial? A. Nominal B. Continuous C. Ordinal D. Ratio

A. Achievement of a goal or threshold - whether it is goal A1c, goal fasting blood glucose, goal blood pressure, or a 5-point reduction in pain - is considered nominal data because it either occurred (YES) or it did not occur (NO). Ordinal data are also categorical data but there is an intrinsic ordering of the categories used to classify the data (e.g., higher to lower, stronger to weaker). Interval data are numerical, and the intervals between each potential data value are equal. The most common example is temperature in degrees Fahrenheit or Celsius. Ratio data are interval data with a natural zero point. For example, time data are ratio data because time 0 is meaningful.

You are seeing a patient, DS, for a 3-month follow-up visit for her hypertension. She is a 46-year-old woman with hypertension, dyslipidemia, and depression. She takes hydrochlorothiazide 25 mg orally daily, lisinopril 40 mg orally daily, and amlodipine 10 mg orally daily. DS currently has a blood pressure of 146/94 mm Hg, heart rate of 70 beats per minute, serum potassium of 4.0 mmol/L, and serum creatinine of 1.2 mg/dL. Which of the following oral drug therapies should be added to help DS achieve her blood pressure goal? A. Spironolactone 12.5 mg once daily B. Hydralazine 10 mg three times daily C. Diltiazem SR 120 mg once daily D. Atenolol 25 mg once daily

A. Adding spironolactone is the best choice for DS. She has a blood pressure goal of less than 130/80 mm Hg, and she is not currently at goal based on the blood pressure measured today (146/94 mm Hg). She has treatment-resistant hypertension (i.e., goal blood pressure not achieved despite receiving three antihypertensive therapies, including a diuretic). Adding spironolactone is the most appropriate choice based on current guidelines for treating resistant hypertension.

Which of the following is the most appropriate conclusion to draw in applying the results of this study to clinical practice? A. E-cigarettes can be considered because they promote abstinence from smoking, although patients continue to use nicotine. B. E-cigarettes should be recommended because they are more effective than nicotine-replacement therapies for smoking cessation. C. E-cigarettes should be recommended because they result in better health outcomes when compared with nicotine-replacement therapies. D. E-cigarettes can be considered because patients appear to prefer them over nicotine-replacement therapies.

A. Although e-cigarette use appears to have resulted in a higher percentage of patients achieving sustained abstinence from tobacco smoking than nicotine replacement therapies, most of the patients in the e-cigarette group were still using e-cigarettes at the end of the study. Thus, one nicotine delivery system was replaced with another. Whether this will lead to better health outcomes (e.g., a lower risk of respiratory diseases and cancer) is unknown because these outcomes were not measured in this study. Also, because patients could use alternative smoking cessation aids after the first 30 days, the improved abstinence rates in the e-cigarette group cannot be attributed with certainty to e-cigarette use alone. It is unknown whether the patients preferred e-cigarettes over nicotine replacement therapies because this variable was not measured. Moreover, the majority of patients who enrolled in the study had previously failed nicotine-replacement therapy, so they were likely to fail to respond to nicotine-replacement therapy again.

AZ is a 64-year-old man who presented to the hospital with non-ST-segment elevation myocardial infarction (NSTEMI) and was taken to the cardiac catheterization laboratory, where a drug-eluting stent was placed in his left circumflex artery. AZ weighs 82 kg, and he is 70 inches tall. Which of the following represents the most appropriate oral dual antiplatelet therapy for AZ upon discharge? A. Aspirin 81 mg daily plus clopidogrel 75 mg daily B. Aspirin 325 mg daily plus ticagrelor 90 mg twice daily C. Aspirin 81 mg daily plus ticagrelor 60 mg twice daily D. Aspirin 325 mg daily plus prasugrel 5 mg daily

A. Aspirin 81 mg daily plus clopidogrel 75 mg daily is the most appropriate dual antiplatelet therapy for AZ because it is consistent with recommendations in the 2014 American College of Cardiology Foundation/American Heart Association guideline for the management of patients with non-ST-elevation acute coronary syndromes.

A patient has an echocardiogram indicating an ejection fraction (EF) of 35% and is planning to initiate the dose-dense portion of the doxorubicin and cyclophosphamide (AC) chemotherapy regimen. Which of the following statements is correct? A. She should receive chemotherapy with a non-anthracycline-containing regimen instead of AC. B. She should proceed with chemotherapy as scheduled with no need to monitor her EF. C. She should proceed with chemotherapy as scheduled, and her EF should be monitored at least once every 3 months. D. She should not receive chemotherapy of any type until her EF improves.

A. Because anthracyclines can cause cardiotoxicity and this patient's ejection fraction is already below the lower limit of the normal range prior to initiating anthracycline-based chemotherapy, anthracycline therapy should be avoided. Patients like this one with an ejection fraction of 45% or less are at increased risk of heart failure and should not receive anthracycline or other cardiotoxic therapy. Cyclophosphamide, methotrexate, and fluorouracil (CMF) is an example of a non-anthracycline-based regimen that is used in patients with breast cancer.

JJ is a 72-year-old female patient who was recently diagnosed with osteoporosis. She comes to your pharmacy asking for a recommendation for a calcium supplement. Her only other medications are levothyroxine 0.075 mg orally once daily for a well-controlled thyroid condition and over-the-counter pantoprazole for heartburn. She admits that her dietary intake of calcium is very limited because of lactose intolerance. Which of the following is the most appropriate calcium supplement and dosing regimen to recommend for JJ (the doses are expressed as elemental calcium)? A. Calcium citrate 600 mg orally twice daily B. Calcium citrate 600 mg orally once daily C. Calcium carbonate 600 mg orally once daily D. Calcium carbonate 600 mg orally twice daily

A. Calcium citrate is a good choice in this older adult patient who takes a proton pump inhibitor, which reduces gastric acidity, because it is a soluble calcium salt and does not require an acidic environment for maximal absorption. The dosage of 600 mg twice daily is sufficient for JJ because the National Osteoporosis Foundation recommends 1200 mg of elemental calcium daily in women over the age of 50.

GT is an 11-year-old male with sickle cell disease and systemic lupus erythematous (SLE) nephritis who receives maintenance immunosuppression with high-dose systemic corticosteroids and recently received a dose of rituximab. He is up to date on his vaccinations through the age of 7 years. His parents have inquired about any vaccinations that he should receive during today's follow-up clinic visit given his comorbidities. Which of the following statements is correct based on recommendations from the Centers for Disease Control and Prevention Advisory Committee on Immunization Practices (ACIP)? A. GT should begin the 2-dose series of meningococcal B vaccine and 2-dose series of 23-valent pneumococcal polysaccharide vaccine (PPSV23); receive tetanus, diphtheria, and acellular pertussis (Tdap); 1 dose of MenACWY-D; and start the 2-dose series for human papillomavirus (HPV) vaccine. B. GT should begin the 3-dose series of meningococcal B vaccine, receive a single dose of 13-valent pneumococcal conjugate vaccine (PCV13), and start the 2-dose series of human papillomavirus (HPV) vaccine C. GT should begin the 2-dose series of meningococcal B vaccine; receive a single dose of 13-valent pneumococcal conjugate vaccine (PCV13) and tetanus, diphtheria, and acellular pertussis (Tdap); and start the 2-dose series of human papillomavirus (HPV) vaccine. D. GT should receive 1 dose of meningococcal B vaccine, begin the 2-dose series of 13-valent pneumococcal conjugate vaccine (PCV13), and start the 2-dose series of human papillomavirus (HPV) vaccine.

A. Given that the patient has not received any vaccines since 7 years of age, he would be due to receive the tetanus, diphtheria, and acellular pertussis (Tdap) and meningococcal vaccines at age 11 based on the ACIP catch-up vaccine schedule. He should receive either the 2- or 3-dose regimen of the meningococcal B vaccine because of his sickle cell disease. He also should also receive a 2-dose series of PPSV23 because he would have already received the PCV13 series prior to age 7 but qualifies for additional coverage with PPSV23 because of his comorbidities. The HPV vaccine would also be indicated given his age since it is indicated to begin at 11-12 years of age with consideration for earlier use as young as 9 years of age in high-risk social situations.

BM is a 67-year-old white male who was admitted with non-ST-segment myocardial infarction (NSTEMI). He was taken to the cardiac catheterization lab for percutaneous coronary intervention (PCI) and received a drug-eluting stent in his left circumflex artery. His past medical history consists of permanent atrial fibrillation, type 2 diabetes mellitus, and hypertension. His creatinine clearance (CrCl) is 64 mL/min. Which of the following oral antithrombotic regimens is most appropriate for BM? A. Rivaroxaban 15 mg daily plus clopidogrel 75 mg daily B. Aspirin 81 mg daily plus prasugrel 10 mg daily C. Edoxaban 60 mg daily plus aspirin 81 mg daily plus clopidogrel 75 mg daily D. Warfarin plus aspirin 81 mg daily plus ticagrelor 90 mg twice daily

A. In the PIONEER AF-PCI trial (Gibson CM et al), patients who had recently undergone PCI with stent placement and required anticoagulation for atrial fibrillation were randomly assigned to receive (1) rivaroxaban 15 mg orally daily + a platelet P2Y12 inhibitor, (2) rivaroxaban 2.5 mg orally twice daily + dual antiplatelet therapy (DAPT), or (3) standard therapy with warfarin + DAPT. The incidence of clinically significant bleeding was significantly lower in the two groups receiving rivaroxaban than the group receiving standard therapy (16.8% [p<0.001] vs. 18% [p<0.001] vs. 26.7%). Therefore, rivaroxaban 15 mg orally daily + clopidogrel 75 mg orally daily is the best choice for BM.

EK is a 4-year-old girl diagnosed with seasonal allergies after developing significant rhinorrhea, sneezing, and conjunctival symptoms when exposed to tree pollen. EK weighs 20 kg and is allergic to penicillin. Which of the following therapies is preferred to treat EK's allergies? A. Fluticasone furoate nasal spray one 27.5-mcg spray in each nostril (55 mcg total) once a day B. Loratadine 5 mg by mouth once a day C. Montelukast 5 mg by mouth once a day D. Azelastine nasal spray one 137-mcg spray in each nostril (274 mcg total) twice a day

A. Intranasal corticosteroids are the treatment of choice for patients less than 6 years of age with seasonal allergic rhinitis. The furoate form of fluticasone is approved for use in patients less than 6 years of age. It also has a low oral systemic bioavailability which is preferred in children to reduce systemic side effects from accidentally swallowing some of the dosage.

After reviewing the chart of a 29-year-old HIV-positive female patient (CD4 count 60 cells/mm3), you notice that her immunization history indicates that she has not received any recommended vaccines since she was 18 years old. Which of the following vaccines should be avoided given her low CD4 count? A. Measles-mumps-rubella B. Tetanus-diphtheria acellular pertussis C. Pneumococcal D. Influenza injectable vaccine

A. Measles-mumps-rubella should be avoided because it is a live, attenuated vaccine that carries a risk for contracting an active infection when given to severely immunocompromised patients (CD4 count <200 cells/mm3) like this one.

Which of the following tenets should be a focal point in the development of a medication reconciliation process to promote safety, regardless of the population served or size of the institution? A. Develop a single shared medication list B. Limit the types of health professionals who may update the medication list C. Specifically target "high-risk" patients with low health literacy for medication reconciliation D. Have pharmacists perform reconciliation at the time of admission and prescribers perform reconciliation at the time of discharge

A. Medication reconciliation is most successful in promoting safety when it is centered around a single list to document the patient's medications. Also referred to as the "one source of truth," this single list should be openly accessible and easily edited by all members of the patient care team.

As you monitor the ASHP and FDA drug shortages websites, you see notice of unavailability of a parenteral drug that you use at your health system. The shortage is expected to last for 6 months, and you have a 4-day supply on hand. Which of the following is the most appropriate and sustainable approach to addressing this problem? A. Work with the pharmacy and therapeutics committee to establish policies for therapeutic interchange. B. Purchase a 3-month supply from your primary supplier or an alternative source. C. Have the patients bring in their own medication. D. Work with case management staff to move or divert patients requiring the drug to another healthcare facility.

A. Once a drug is listed on a drug shortages list, it is widely unavailable. Because this parenteral drug will not be available for at least 6 months, you need to be proactive to avoid lapses in therapy. Establishing policies for therapeutic interchange is the most appropriate and sustainable approach to addressing the shortage problem.

Which of the following prophylactic antiemetic therapies should a patient receive prior to his/her dose of paclitaxel? A. Ondansetron 8 mg orally 30 minutes prior to chemotherapy B. Ondansetron 8 mg orally and dexamethasone 12 mg orally 30 minutes prior to chemotherapy C. Ondansetron 8 mg orally and fosaprepitant 150 mg IV 30 minutes prior to chemotherapy D. No prophylactic antiemetic therapy is required before paclitaxel

A. Paclitaxel is a chemotherapeutic agent with low emetogenicity. Authoritative guidelines recommend a serotonin 5-HT3 antagonist, such as ondansetron, or dexamethasone as single-agent prophylactic antiemetic therapy.

TT is 55-year-old man with osteoarthritis and a history of coronary artery disease who presents for further management of his osteoarthritis pain. He has been taking acetaminophen 500-1000 mg orally four times a day with no improvement in osteoarthritis pain for the past 3 months. His other current medications include atorvastatin 40 mg orally daily and amlodipine 5 mg orally daily. Which of the following represents the most appropriate alternative to acetaminophen for osteoarthritis pain management in TT at this time? A. Naproxen sodium 220 mg twice daily B. Celecoxib 200 mg twice daily C. Diclofenac 50 mg twice daily D. Ibuprofen 600 mg three times daily

A. Population-based studies evaluating the influence of the type of nonsteroidal anti-inflammatory drug (NSAID) and dosage on cardiovascular outcomes suggest that low-dose naproxen (500 mg/day or less) is associated with a lower risk for adverse cardiovascular outcomes than other NSAIDs. The Precision trial (Nissen et al) demonstrated that celecoxib was non-inferior to naproxen and ibuprofen with regards to cardiovascular safety with fewer adverse effects, but the naproxen dosage was 375 mg twice daily, not 220 mg twice daily. Therefore, naproxen sodium 220 mg twice daily is the most appropriate NSAID regimen for this patient.

LK had severe nausea and vomiting from her doxorubicin and cyclophosphamide (AC) and was admitted for dehydration and electrolyte depletion. On medication reconciliation at the time of admission, the patient stated that she was taking oxycodone 5 mg orally as needed for pain and using an average of 5 doses daily. Her pain was previously well controlled, but she has not been able to take her oxycodone for the past 1.5 days due to vomiting. The team wants to change her pain regimen to intravenous (IV) morphine by patient-controlled analgesia (PCA). Which of the following initial morphine PCA rates should be recommended for this patient? A. 0.5 mg/hr B. 1 mg/hr C. 1.5 mg/hr D. 3 mg/hr

A. Prior to her vomiting, LK was using a total of 25 mg of oxycodone daily (five 5-mg doses). Converting this to oral morphine using the equianalgesic ratio of oxycodone 20 mg = morphine 30 mg, the total oral daily dose of morphine would be 37.5 mg. This oral dose should then be converted to the IV formulation of morphine using the ratio of oral morphine 30 mg = IV morphine 10 mg, resulting in a daily IV morphine dose of 12.5 mg. The infusion rate of 0.5 mg/hr is determined by dividing the 12.5 mg/day IV dose by 24 hours/day. No dose reduction is made for cross-tolerance due to the patient's poor pain control.

AB is a 65-year-old African-American woman admitted to the cardiac care unit (CCU) for New York Heart Association class IV acute decompensated heart failure. Physical examination reveals warm extremities and rales upon auscultation. Allergies: NKDA PMHx: non ST-segment elevation myocardial infarction with stenting (6 months ago), hypertension, osteoarthritis, bilateral renal artery stenosis Home MedicationsAmlodipine 5 mg orally dailyCarvedilol 25 mg orally twice dailyClopidogrel 75 mg orally dailyDiclofenac 75 mg orally twice dailyIsosorbide dinitrate 40 mg orally three times dailyPravastatin 20 mg orally daily Vital signs: BP 138/85 mm Hg, HR 92 bpm, RR 24/min, Ht 5'3", Wt 150 lb Pertinent Labs & Diagnostics: Na = 139 mEq/L K = 5.1 mEq/L Cl = 105 mEq/L HCO3 = 25 mEq/LBUN = 18 mg/dL SCr = 1.1 mg/dL Glucose = 84 mg/dLPCWP = 25 mm Hg, Cardiac index = 3 L/min/m2 Echocardiogram: left ventricular systolic function appears severely depressed Visually estimated ejection fraction = 20% The interprofessional team has been debating whether to prescribe eplerenone for AB. Which of the following would be a critical factor to consider in making this decision in the case of AB? A. Serum potassium level B. Blood pressure C. Serum creatinine level D. Presence of renal artery stenosis

A. Serum potassium level is correct because her measured level is 5.1 mEq/L, and the use of eplerenone is not recommended in patients with serum potassium levels above 5.0 mEq/L.

MK is a 39-year-old HIV-positive Caucasian man who started taking the "quad pill" (elvitegravir + cobicistat + tenofovir disoproxil fumarate + emtricitabine) about 4 weeks ago. He has been feeling very sick over the last week and complains of inability to urinate. The physician thinks that MK has developed Fanconi syndrome, which is characterized by tubular proteinuria, aminoaciduria, phosphaturia, glycosuria, and bicarbonate wasting. Which of the following is the most likely cause of his acute kidney injury? A. Tenofovir disoproxil fumarate B. HIV-associated nephropathy (HIVAN) C. Cobicistat D. Emtricitabine

A. Tenofovir is the drug that has been associated with Fanconi syndrome. A newer prodrug, tenofovir alafenamide, is associated with a much lower risk of this adverse effect than tenofovir disoproxil fumarate.

Which of the following statements regarding breast cancer screening is TRUE? A. Routine screening is recommended beginning at age 40-45 years B. Routine screening should be discontinued in women 65 years of age and older C. Breast self-exam is recommended for women 25-39 years of age D. Breast magnetic resonance imaging (MRI) is the preferred imaging modality

A. The American Cancer Society (ACS), the U.S. Preventive Services Task Force (USPSTF), and National Comprehensive Cancer Network (NCCN) each have guidelines for breast cancer screening. The NCCN recommends routine mammography beginning at age 40, and ACS recommends it starting at age 45. The USPSTF recommends routine screening beginning at age 50, but also makes a qualified statement that screening may begin at age 40 if the risks and benefits have been considered and discussed.

You have been asked to participate on an interprofessional team to review admission orders for patients transferred to your health system from other hospitals. Which of the following is identified by the Centers for Medicare and Medicaid Services (CMS) as a hospital-acquired condition that you should document at the time of patient admission to your facility for reimbursement purposes? A. Stage III or IV pressure ulcer B. Inappropriate antibiotic therapy C. Heart failure diagnosis D. Use of more than five chronic medications

A. The CMS has established a list of hospital-acquired conditions that could reasonably have been prevented through the application of evidence-based guidelines. Receiving hospitals can document the presence of these conditions at the time of admission so that their reimbursement from CMS for the patient stay is not denied. Stage III or IV pressure ulcers are among the hospital-acquired conditions that you should document at the time of admission of patients so that reimbursement by CMS is not withheld.

The Hospital Consumer Assessment of Healthcare Providers and Systems (HCAHPS) is designed to: A. Measure patient perception of patient care provided B. Detect areas of health care that are not compliant with standards C. Identify population health trends for targeted disease states D. Protect patient rights during health care

A. The HCAHPS is designed to measure patient perception of patient care provided through use of a questionnaire with 32 standard questions. It can be used for assessing trends in individual organizations and comparing responses among different organizations.

The Joint Commission and other accreditation organizations have incorporated antimicrobial stewardship requirements into their standards. You have been asked to oversee the education of practitioners about antimicrobial resistance and stewardship practices. When must education be provided to licensed independent practitioners in your Joint Commission-accredited organization? A. Upon hire or granting of initial privileges and periodically thereafter based on organizational needs B. Upon hire or initial credentialing C. At least annually D. If needed as determined by the medical staff executive committee.

A. The Joint Commission standards require practitioner education about antimicrobial resistance and stewardship practices upon hire or granting initial privileges and periodically thereafter based on organizational needs.

Which of the following best characterizes the United States Pharmacopeia (USP)? A. Standard-setting organization B. Enforcement agency C. Accreditation organization D. Component of the Food and Drug Administration (FDA)

A. The USP sets standards. It is up to regulatory bodies and accreditation organizations to incorporate the USP standards into their regulations and accreditation standards, respectively.

RB is a 77-year-old man with a history of Parkinson's disease, osteoporosis, and hypertension. His current medication regimen includes trihexyphenidyl, carbidopa/levodopa, risedronate, lisinopril, and low-dose aspirin. In performing a chart review, you note that his pulse has been consistently greater than 95 beats per minute (range 92 to 116 beats per minute). Which of the following would be the most appropriate drug treatment recommendation for RB at this time? A. Discontinue trihexyphenidyl and add rasagiline B. Discontinue lisinopril and add metoprolol C. Discontinue carbidopa/levodopa and add rotigotine D. Discontinue risedronate and add denosumab

A. The anticholinergic effects of trihexyphenidyl are substantial and commonly manifest as tachycardia. This medication is not suitable for a geriatric patient because of the anticholinergic burden. Discontinuing trihexyphenidyl and switching to the monoamine oxidase inhibitor rasagiline is the most appropriate choice for this patient.

Based on these results, which of the following is the most appropriate conclusion that can be drawn regarding the efficacy of DRUG A for preventing CV events in patients with type 2 diabetes who are at a high risk of CV events? A. DRUG A is superior to placebo B. DRUG A is inferior to placebo C. DRUG A is no worse than placebo D. DRUG A is no better than placebo

A. The confidence interval (CI) spanning from 0.56 to 0.78 demonstrates the superiority of DRUG A in efficacy compared with placebo because the CI does not touch or include the line of unity (i.e., 1). The result also demonstrates that the difference in efficacy between the two treatment groups is significant.

You are asked to counsel a patient about potential drug interactions with his new HIV antiretroviral medications: bictegravir + tenofovir alafenamide + emtricitabine. His other medications are a multivitamin and mineral formulation, acetaminophen as needed, and a fluticasone nasal inhaler. Which of these agents (if any) could reduce the efficacy of antiretroviral therapy in this patient? A. The multivitamin and mineral formulation B. Acetaminophen C. Fluticasone nasal inhaler D. None of the agents can affect the antiretroviral therapy

A. The multivitamin and mineral formulation could reduce the efficacy of antiretroviral therapy in this patient because it contains polyvalent cations (aluminum, magnesium, calcium, and iron), which can cause chelation and decreased absorption of bictegravir. Bictegravir should be taken at least 2 hours before or 6 hours after polyvalent cations to minimize this interaction.

What type of data is the primary outcome in the REDUCE-IT study? A. Nominal B. Continuous C. Ordinal D. Ratio

A. The primary outcome occurred when one of five discrete events occurred. Discrete events are nominal data because an event (e.g., nonfatal stroke) falls in a mutually exclusive yes or no category (yes, the patient had a nonfatal stroke or no, the patient did not have a nonfatal stroke). The results are the expressed as a percentage of patients who had at least one of the five prespecified outcome events.

MR, an 11-year-old Vietnamese female who uses complementary/alternative medicine as well as traditional medications, presents to the asthma clinic for reevaluation of her asthma action plan with her aunt who primarily speaks Vietnamese. To provide culturally competent care to MR and her aunt, the clinic should: A. Be open and center communication on the patient's and her aunt's cultural beliefs, which could affect adherence B. Take into consideration MR's developmental stage and her and her aunt's preferred language when providing counseling C. Discourage MR's aunt from participating in MR's care to avoid medication errors D. Avoid asking potentially sensitive questions regarding complementary and alternative medicine interventions that the aunt may be using for MR

B. According to Standard 1 of the National Standards for Culturally and Linguistically Appropriate Services in Health and Health Care, effective care should be provided in a manner compatible with a patient's and caregiver's preferred language. The American Academy of Pediatrics recommends taking into consideration the developmental stage when providing counseling to pediatric patients.

AF is a 13-year-old male patient with moderate persistent asthma that previously was well controlled with albuterol HFA 90 mcg/inhalation by metered-dose inhaler (MDI) 2 inhalations every 6 hours as needed for wheezing or shortness of breath, fluticasone 100 mcg/salmeterol 50 mcg HFA MDI 2 inhalations twice daily, and montelukast 10 mg orally daily at bedtime. He recently joined the cross-country team at his junior high school and has been experiencing coughing and shortness of breath after 2 hours of practice almost daily. He would like some additional information regarding his asthma management. What updates to his asthma management plan would you provide at this time? A. Advise the patient that because his asthma is no longer well controlled he should discontinue his activity with the cross country team B. Review the asthma action plan and adherence with the patient; add albuterol HFA 90 mcg/inhalation MDI 2 inhalations 30 minutes prior to cross country practice C. Review the asthma action plan and adherence with the patient; increase his fluticasone 100 mcg/salmeterol 50 mcg HFA MDI to 4 inhalations twice daily during the cross country season D. Review the asthma action plan with the patient and assess his inhaler technique

B. All patients with asthma should have an asthma action plan that is used to help them better manage their symptoms and lessen the severity of exacerbations. Increased physical activity can be a trigger for asthma symptoms, which can be easily prevented by using a short-acting beta2-agonist (i.e., albuterol) prior to exercise.

LM is a 63-year-old woman who presents to the hospital with ST-segment elevation myocardial infarction (STEMI). She has a past medical history significant for hyperlipidemia and active cigarette smoking. She is taken to the cardiac catheterization laboratory for percutaneous coronary intervention (PCI), and two drug-eluting stents (DES) are placed during the procedure. Which of the following is the most appropriate lipid-lowering therapy for LM? A. Rosuvastatin 10 mg daily B. Atorvastatin 80 mg daily C. Simvastatin 40 mg daily D. Pravastatin 40 mg daily

B. Atorvastatin 80 mg daily is appropriate because the 2013 American College of Cardiology (ACC) Foundation/American Heart Association (AHA) STEMI guidelines recommend a high-intensity statin in all STEMI patients (assuming there are no contraindications to use). The 2013 ACC/AHA Guideline on the Treatment of Blood Cholesterol defines high-intensity statin therapy as atorvastatin 40-80 mg daily or rosuvastatin 20-40 mg daily.

Based on the description of the REDUCE-IT trial above, which of the following is the best interpretation of the study results? A. Treatment with icosapent ethyl significantly reduced the risk of CV death, nonfatal myocardial infarction, nonfatal stroke, coronary revascularization, and unstable angina when compared with placebo. B. After 4.9 years of treatment, patients assigned to the icosapent ethyl group were significantly less likely to die from CV causes than patients assigned to the placebo group. C. Any benefits derived from icosapent ethyl treatment were entirely offset by an increase in hospitalizations due to atrial fibrillation or flutter. D. It is not possible to determine whether icosapent ethyl significantly reduced the incidence of the primary endpoint when compared with placebo because a p-value is not reported.

B. Because this is an intention-to-treat analysis, we can only state (with certainty) that results are based on the assigned treatment group - not the actual use of the treatment since a patient assigned to icosapent ethyl (or placebo) may or may not have taken the assigned treatment. A per-protocol analysis could be used to determine whether actual use of icosapent ethyl resulted in reductions in the composite outcome or individual components, such as CV mortality. A 95% confidence interval (CI) that does not touch or cross the number "1" (i.e., the line of no effect) is indicative of a statistically significant difference. In this trial, for CV mortality, the 95% CI 0.66-0.99 did not touch or cross 1, and it corresponds to a p-value less than 0.05. Therefore, the difference between icosapent ethyl and placebo in CV mortality after a median follow-up time of 4.9 years is statistically significant.

CW, an 83-year-old male, presents to the emergency department (ED) with his wife who states that he has been slurring his words for the past 2-½ hours and has right-sided weakness. CW has a past medical history of hypertension (well-controlled by his current medication regimen) and gout. His current medications include aspirin 81 mg orally once daily, atenolol 50 mg orally once daily, hydralazine 50 mg orally three times daily, and a multivitamin orally once daily. His vital signs on ED arrival were a blood pressure of 156/87 mm Hg, pulse of 87 beats per minute, respiratory rate of 15 breaths per minute, and temperature of 37°C. Your team has asked you to assess the patient for eligibility for tissue plasminogen activator (tPA). Which of the following is your assessment? A. CW is not eligible for tPA because of his high blood pressure B. CW is eligible for tPA and should receive it C. CW is not eligible for tPA because of his use of aspirin D. CW is not eligible for tPA because of his advanced age

B. CW is a candidate for tissue plasminogen activator (tPA), despite meeting multiple exclusion criteria in guidelines from the American Heart Association and American Stroke Association. He presented within the required 3-hour window (if CW had presented after 3 hours, he would not have been eligible for the expanded treatment window of 4-½ hours because he is older than 80), and his blood pressure is < 185/110 mm Hg. Aspirin use is not a contraindication to the use of tPA.

TS is a 70-year-old woman who was seen in a community mental health clinic. She was sent to you for follow-up monitoring of her antidepressant therapy, citalopram 20 mg orally once daily. In addition to subjective monitoring to determine efficacy for reducing depression symptoms, which of the following objective tests should be performed in TS at regular intervals? A. Brain magnetic resonance imaging (MRI) B. Electrocardiogram (ECG) C. Serum calcium concentration D. Liver panel

B. Citalopram can cause potentially fatal QT interval prolongation and torsade de pointes, so an ECG should be performed periodically in patients receiving the drug. The maximum recommended dosage for patients like TS who are more than 60 years of age is 20 mg/day because exposure to the drug is increased in this age group compared with younger patients, which may increase the risk of QT interval prolongation.

Which of the following is a consideration in determining whether a risk evaluation and mitigation strategy (REMS) program is needed for a drug? A. Cost-to-benefit ratio to administer the REMS program for a drug or treatment B. Expected risk and benefit with respect to the condition treated C. Manufacturer liability D. Patient satisfaction

B. Considerations in determining the need for a REMS program include the estimated size of the population likely to use the drug, seriousness of the disease or condition for which the drug will be used, expected benefit from the drug with respect to the disease or condition, expected or actual duration of treatment with the drug, seriousness of any known or potential adverse events related to use of the drug and background rate of such events in the population likely to use the drug, and whether the drug is a new molecular entity.

AB is a 65-year-old African-American woman admitted to the cardiac care unit (CCU) for New York Heart Association class IV acute decompensated heart failure. Physical examination reveals warm extremities and rales upon auscultation. Allergies: NKDA PMHx: non-ST-segment myocardial infarction with stenting (6 months ago), hypertension, osteoarthritis, bilateral renal artery stenosis Home Medications Amlodipine 5 mg orally dailyCarvedilol 25 mg orally twice dailyClopidogrel 75 mg orally dailyDiclofenac 75 mg orally twice dailyIsosorbide dinitrate 40 mg orally three times dailyPravastatin 20 mg orally daily Vital signs: BP 138/85 mm Hg, HR 92 bpm, RR 24/min, Ht 5'3", Wt 150 lb Pertinent Labs & Diagnostics:Na = 139 mEq/L K = 5.1 mEq/L Cl = 105 mEq/L HCO3 = 25 mEq/LBUN = 18 mg/dL SCr = 1.1 mg/dL Glucose = 84 mg/dLPCWP = 25 mm Hg, Cardiac index = 3 L/min/m2 Echocardiogram: left ventricular systolic function appears severely depressedVisually estimated ejection fraction = 20% Which of the following infusions should be initiated first to address acute decompensated heart failure in AB? A. Dobutamine B. Furosemide C. Milrinone D. Nitroglycerin

B. Furosemide is correct because this patient is fluid overloaded with adequate perfusion (acute decompensated heart failure subset II) and will respond best to removal of the fluid using a loop diuretic.

RP is a 65-year-old community-dwelling female who recently became eligible for Medicare. She is interested in selecting a health insurance plan that prioritizes quality care, noting "I want to live to be a healthy 100!" Which of the following tools or organizations could she use to find a plan that outperforms the others on quality of care measurements? A. The Centers for Medicare and Medicaid Services (CMS) Partnership for Patients B. The Healthcare Effectiveness Data and Information Set (HEDIS) C. The Joint Commission D. America's Health Insurance Plans (AHIP)

B. HEDIS is a tool utilized by health plans to measure performance in providing quality care and services. It can be utilized to compare quality measures and domains of care for health insurance companies on an "apples-to-apples" basis. HEDIS is the best choice for RP.

Which of the following is a possible patient-related barrier to adherence to endocrine therapy for cancer? A. Arthralgias and myalgias B. Depression C. Nausea D. Poor relationship with the oncologist

B. History of depression is a potential patient-related barrier to adherence to endocrine therapy.

LK is a patient with breast cancer who is receiving a chemotherapy regimen consisting of doxorubicin and cyclophosphamide. She called the nurse to report a fever, and she presented in clinic with hypotension and tachycardia. Blood work revealed acute renal failure (creatinine clearance 45 mL/min) in addition to neutropenia. Which of the following is the most appropriate empiric antibiotic regimen for LK? A. Cefepime 2 g IV every 8 hours B. Cefepime 2 g IV every 12 hours C. Ciprofloxacin 750 mg IV every 12 hours D. Ciprofloxacin 500 mg orally every 8 hours and amoxicillin/clavulanate 500 mg orally every 8 hours

B. LK is considered to be a high-risk patient with neutropenia due to hypotension, tachycardia, and renal failure. High-risk patients should be admitted to the hospital and receive intravenous antimicrobial treatment. Cefepime monotherapy is appropriate for empiric treatment of febrile neutropenia in patients like LK with cancer. It is usually given every 8 hours, but because of her new-onset renal dysfunction, administering the drug every 12 hours is more appropriate for LK.

JH is a 60-year-old woman seeing you on a return visit to clinic today. She has a past medical history of hypertension, diabetes mellitus, and chronic kidney disease (CKD). Her albumin/creatinine ratio is 40 mg/g, and she has an estimated glomerular filtration rate (eGFR) of 65 mL/min/1.73 m2. How would you classify this patient's eGFR and proteinuria based on the 2012 Kidney Disease: Improving Global Outcomes (KDIGO) guidelines and her albumin/creatinine ratio? A. Normal eGFR and no proteinuria B. Mildly decreased eGFR and moderately increased proteinuria C. Mildly decreased eGFR and no proteinuria D. Moderately decreased eGFR and moderately increased proteinuria

B. Mildly decreased eGFR and moderately increased proteinuria is correct based on JH's eGFR of 65 mL/min/1.73 m2 and albumin/creatinine ratio of 40 mg/g.

Which of the following statements best describes a potential threat to internal or external validity of the study of e-cigarettes vs. nicotine-replacement therapies for smoking cessation? A. The internal validity of the study is threatened because the investigators provided the initial supply of medication. B. The internal validity of this study is threatened because participants were predisposed to failing nicotine-replacement therapy. C. The external validity of the study is threatened because participants could select whatever nicotine-replacement product they wished to use. D. The external validity of the study is threatened because the investigators and participants were not blinded to the treatment assignment.

B. Most smokers make multiple quitting attempts before successfully achieving sustained abstinence. The majority (75%) of study participants had tried nicotine-replacement therapy unsuccessfully, so the study sample included a disproportionately high number of patients who had previously failed nicotine-replacement therapy. Thus, the internal validity is adversely impacted because one treatment (nicotine-replacement) is inherently less likely to be effective, which could exaggerate the magnitude of the benefit from e-cigarette use.

Which of the following is the most appropriate interpretation of these results? A. If 24 patients who are at a high risk for CV disease take DRUG A to treat diabetes mellitus for 1 year, one CV death, nonfatal MI, or nonfatal stroke will be avoided. B. If you treat 24 patients with diabetes mellitus who are at a high risk of CV disease for approximately 3 years with DRUG A, one CV death, nonfatal MI, or nonfatal stroke will be avoided. C. If 1000 people who are at a high risk for CV disease take DRUG A to treat diabetes mellitus, 42 CV events (fatal and nonfatal) will be avoided. D. If you treat 1000 people with diabetes mellitus who are at a high risk for CV disease for 5 years with DRUG A, there will be a 69% decrease in the number of CV events.

B. Provided there is a statistically significant difference between treatment groups, the number needed to treat (NNT) indicates the number of patients you would need to treat with DRUG A to prevent one event over the study time frame. In this example with an NNT of 24, if you treated 24 patients with diabetes mellitus who are at a high risk of CV disease with DRUG A for approximately 3 years, 1 fewer event (i.e., CV death, nonfatal MI, or nonfatal stroke) would be expected.

A pharmacist works at a rural critical access hospital with only 20 beds. The hospital has a very limited medical library with a few medical journals and textbooks but Internet access is available at most work stations. SL is a 73-year-old Asian woman who was admitted to the hospital. While gathering the patient's medication history, she stated that about a month ago she started a new once-weekly injectable drug therapy for the treatment of type 2 diabetes mellitus. She has given herself four doses so far. When questioned about how well the new treatment is working, she states that after the third dose she started to experience an "itchy sensation" that has intensified over the past several days. The patient's skin and whites of her eyes appear jaundiced. You suspect that the new medication may be the culprit. The Food and Drug Administration (FDA)-approved prescribing information for the drug makes no mention of liver injury. Which of the following sources would be the most helpful to determine whether this medication can cause acute liver injury? A. The Drug Information Handbook B. The PubMed search engine C. The manufacturer's website D. The UpToDate database

B. PubMed is a free search engine that provides online access to the MEDLINE database created and maintained by the National Library of Medicine. The database includes citations related to life sciences and biomedical journals and conference abstracts, most of which are published in English and from North American sources. PubMed is one of the most comprehensive databases in the world, although other databases (e.g., EMBASE) cover additional journals available in Europe. A PubMed search would be an excellent place to start to see if this adverse reaction has been reported by other clinicians. A search of the FDA Adverse Events Reporting System (FAERS) may also be worthwhile, but FAERS is difficult to navigate and searches are far more time consuming than PubMed searches. PubMed is the best place to start.

You are seeing a patient, BB, for his annual visit in clinic today for a blood pressure follow up. He is a 48-year-old man with hypertension, diabetes mellitus, dyslipidemia, and chronic kidney disease. BB also is a current smoker. His blood pressure is 128/78 mm Hg today. He reports getting the influenza vaccine last week at his local pharmacy. Which of the following immunizations is appropriate for BB today based on recommendations from the Centers for Disease Control and Prevention Advisory Committee on Immunization Practices (ACIP)? A. Human papillomavirus (HPV) B. 23-valent pneumococcal polysaccharide vaccine(PPSV-23) C. Zoster recombinant (RZV) D. Hepatitis B (Hep B)

B. The 23-valent pneumococcal polysaccharide vaccine is indicated for this 48-year-old patient because he is a smoker and has diabetes (a chronic medical condition).

You are working with an antimicrobial stewardship committee to develop a guideline for management of acute otitis media (AOM) at a health system that consists of outpatient physician offices and a 300-bed community hospital. One of the committee's goals is to decrease antibiotic use without compromising patient safety. Which of the following initial strategies would be consistent with evidence-based recommendations for the treatment of AOM? A. Recommend a 3-day course of antibiotic therapy for AOM in children less than 2 years of age B. Use observation instead of antibiotic therapy for 48-72 hr for non-toxic appearing children more than 2 years of age with bilateral AOM without otorrhea C. Recommend a 5-day course of antibiotics for AOM in children 2 to 5 years of age D. Use observation instead of antibiotic therapy for 48-72 hr for non-toxic appearing children less than 2 years of age with unilateral AOM and otorrhea

B. The American Academy of Pediatrics AOM clinical practice guideline suggests observation for 48-72 hours as an alternative to antibiotic therapy only in patients who are more than 2 years of age with unilateral or bilateral AOM without otorrhea.

In a triple-blind clinical trial, patients with permanent or paroxysmal atrial fibrillation or atrial flutter using warfarin for at least 3 months who were undergoing an elective invasive procedure requiring interruption of warfarin therapy were randomly assigned to receive no bridging with subcutaneous placebo (n=950) or bridging with dalteparin 100 units/kg subcutaneously twice daily (n=934) from 3 days until 24 hours before the procedure and for 5 to 10 days after the procedure until the international normalized ratio was at least 2. The incidence of the primary outcome of major bleeding was 1.3% in the no bridging group compared with 3.2% in the bridging group; RR 0.41 (0.20 to 0.78), p<0.005. What is the absolute change in the risk for major bleeding when bridging therapy is given when compared with no bridging therapy based on the results of this trial? A. 1.46% B. 1.9% C. 2.46 D. 53

B. The absolute risk difference is calculated by subtracting the event rate in the intervention group (i.e., bridging) from the event rate in the control group (i.e., no bridging): 1.3%-3.2% =1.9%.

What is the absolute difference in the risk of a primary outcome event when comparing the icosapent ethyl and placebo groups in the REDUCE-IT trial? A. 0.75% B. 4.8% C. 17.2% D. 25%

B. The absolute risk difference is calculated by subtracting the event rate in the intervention group (i.e., icosapent ethyl) from the event rate in the control group (i.e., placebo) group: |17.2%-22.0%|= 4.8%.

In the multicenter, randomized, open-label, phase 3 noninferiority AWARD-6 trial, patients with type 2 diabetes currently receiving metformin were randomly assigned to receive either once-weekly dulaglutide (n=299) or once-daily liraglutide (n=300). The primary outcome was change from baseline in A1c at 26 weeks, and a mean difference of < 0.4% was set as the noninferiority margin. The change in A1c from baseline to week 26 was -1.42% in the dulaglutide group and -1.36% in the liraglutide group, with a mean difference of -0.06% (95%CI -0.19% to 0.07%). Which of the following is the most appropriate interpretation based solely on the efficacy results of the AWARD-6 trial? A. Dulaglutide should be recommended over liraglutide for patients with type 2 diabetes treated with metformin due to superior A1c reduction B. Dulaglutide is no worse than liraglutide for reducing A1c in patients with type 2 diabetes treated with metformin C. Dulaglutide and liraglutide are equally effective for reducing A1c in patients with type 2 diabetes treated with metformin D. An appropriate interpretation of the AWARD study findings cannot be made without a p-value

B. The confidence interval does not cross or touch the noninferiority margin of < 0.4%, indicating that the criteria for noninferiority are met. Therefore, dulaglutide is not inferior to (i.e., no worse than) liraglutide. Superiority cannot be claimed because the results of superiority testing are not reported. The treatments may or may not be equal in efficacy- this was not an equivalence study and superiority is unknown. A p-value is not required to judge whether the 95% CI exceeded the noninferiority margin.

SB is a 55-year-old Caucasian male who presents to clinic today for a follow-up visit after a 4-day hospitalization for newly-diagnosed heart failure. He is still complaining of dyspnea on ambulation. He has a past medical history of chronic kidney disease and hypertension along with his newly-diagnosed heart failure. His BP is 128/87 mm Hg, HR is 78 bpm, creatinine clearance (CrCl) is 25 mL/min, and ejection fraction is 30%. His current drug regimen includes lisinopril 2.5 mg orally daily, bisoprolol 1.25 mg orally daily, bumetanide 1 mg orally twice daily, and eplerenone 25 mg orally daily. Which of the following is the first change that should be made to SB's medication regimen at this time? A. Increase the dosage of lisinopril from 2.5 mg orally daily to 5 mg orally daily B. Discontinue eplerenone 25 mg orally daily C. Reduce the dosage of bumetanide from 1 mg orally twice daily to 1 mg orally once daily D. Stop lisinopril 2.5 mg orally daily and start sacubitril/valsartan 49 mg/51 mg orally twice daily

B. The use of aldosterone antagonists, such as eplerenone, could be harmful in this patient due to life-threatening hyperkalemia and renal insufficiency. Because this patient's CrCl is less than 30 mL/min, eplerenone should be stopped at this time.

AM is a 61-year-old woman with a history of coronary artery disease, type 2 diabetes mellitus, and hyperlipidemia who was recently admitted and received a drug-eluting stent in her left-anterior descending artery for management of ST segment-elevation myocardial infarction (STEMI). Her post-procedure ejection fraction is 25%. Which of the following represents the most appropriate duration of dual antiplatelet therapy (DAPT) for AM? A. 1 month B. 30 months C. 6 months D. 12 months

B. Thirty months is correct. This patient has a DAPT score of 4 points based on her type 2 diabetes, MI at presentation, and reduced ejection fraction, which suggests that the risk for recurrent thrombotic events is higher than the risk for major bleeding. Therefore, she would benefit from prolonged DAPT beyond 12 months.

A 25-year-old HIV-positive woman has a CD4 count of 50 cells/mm3. She has just started antiretroviral therapy and trimethoprim-sulfamethoxazole one double-strength tablet orally once daily. Given her frequent exposure to cats, a toxoplasma IgG test was ordered and the result was positive. Should she receive additional prophylactic therapy for toxoplasmosis encephalitis? A. No, she is not at risk for toxoplasmosis encephalitis B. No, she is already protected from toxoplasmosis encephalitis by her trimethoprim-sulfamethoxazole C. Yes, she should receive fluconazole 400 mg orally once daily D. Yes, she should receive dapsone 50 mg orally once daily

B. This patient is already protected from toxoplasmosis encephalitis by trimethoprim-sulfamethoxazole, which is the preferred prophylaxis agent for toxoplasmosis encephalitis, so additional prophylaxis is not needed.

AZ is a 64-year-old man who presented to the hospital with non-ST-segment elevation myocardial infarction (NSTEMI) and was taken to the cardiac catheterization laboratory. AZ weighs 82 kg, and he is 70 inches tall. Once arterial access was obtained, he was initiated on intravenous (IV) tirofiban. Which of the following is the most appropriate anticoagulant therapy for AZ at this time? A. Enoxaparin B. Unfractionated heparin C. Bivalirudin D. Fondaparinux

B. Unfractionated heparin (UFH) is the best choice for AZ because in the 2014 American College of Cardiology Foundation/American Heart Association guideline for the management of patients with non-ST-elevation acute coronary syndromes, UFH is recommended for patients like AZ who are receiving a glycoprotein IIb/IIIa inhibitor (tirofiban).

RZ is a 43-year-old man who was diagnosed with non-ST-segment myocardial infarction 1 month ago. He underwent percutaneous coronary intervention with stenting and was discharged on appropriate medical therapy. He is now presenting to his primary care physician's office 1 month after smoking cessation. He quit "cold turkey," and he is now fearful that he will resume smoking cigarettes. Which of the following smoking cessation therapies is most appropriate for RZ at this time? A. Nicotine patch B. Varenicline C. Bupropion D. Clonidine

B. Varenicline is the most appropriate choice for RZ based on the clinical practice guideline from the American Thoracic Society for initiating pharmacologic treatment in tobacco-dependent adults (Leone et al).

AB is a 65-year-old African-American woman admitted to the cardiac care unit (CCU) because of New York Heart Association class IV acute decompensated heart failure. Physical examination reveals warm extremities and rales upon auscultation. Allergies: NKDA PMHx: non ST-segment elevation myocardial infarction with stenting (6 months ago), hypertension, osteoarthritis, bilateral renal artery stenosis Home MedicationsAmlodipine 5 mg orally dailyCarvedilol 25 mg orally twice dailyClopidogrel 75 mg orally dailyDiclofenac 75 mg orally twice dailyIsosorbide dinitrate 40 mg orally three times dailyPravastatin 20 mg orally daily Vital signs: BP 138/85 mm Hg, HR 92 bpm, RR 24/min, Ht 5'3", Wt 150 lb Pertinent Labs & Diagnostics:Na = 139 mEq/L K = 5.1 mEq/L Cl = 105 mEq/L HCO3 = 25 mEq/LBUN = 18 mg/dL SCr = 1.1 mg/dL Glucose = 84 mg/dLPCWP = 25 mm Hg, Cardiac index = 3 L/min/m2 Echocardiogram: left ventricular systolic function appears severely depressedVisually estimated ejection fraction = 20% Which of AB's medications most likely contributed to her symptoms and should be discontinued? A. Amlodipine B. Carvedilol C. Diclofenac D. Isosorbide dinitrate

C. AB is fluid overloaded with adequate perfusion (acute decompensated heart failure subset II). Nonsteroidal anti-inflammatory drugs (NSAIDs) decrease renal blood flow leading to sodium and water retention, expanded intravascular volume, and increased systemic vascular resistance. Therefore, the NSAID diclofenac needs to be stopped to allow AB's soon-to-be-prescribed diuretics to work.

LK called the clinic to report breakthrough nausea and vomiting with two episodes of emesis (although she denies dehydration) after receiving dose-dense doxorubicin and cyclophosphamide (AC) yesterday. She received antiemetic prophylaxis with a neurokinin-NK1 antagonist (fosaprepitant 150 mg IV on day 1), serotonin 5-HT3 antagonist (ondansetron 8 mg IV on day 1), corticosteroid (dexamethasone 12 mg IV on day 1, followed by 8 mg orally daily on days 2-4), and olanzapine (10 mg orally on day 1 followed by 10 mg orally daily on days 2-4). Which of the following interventions should be utilized to treat her breakthrough nausea and vomiting? A. Add aprepitant 80 mg orally once daily B. Add palonosetron 0.25 mg IV X 1 in clinic C. Add prochlorperazine 10 mg orally every 6 hours D. Increase her oral dexamethasone dose to 10 mg twice daily

C. Adding prochlorperazine is appropriate because it provides an additional mechanism of action (antidopaminergic activity) to combat nausea/vomiting. Adding a phenothiazine is supported by antiemetic guidelines.

RD is an 83-year-old male who recently had a cardioembolic stroke. He has a history of atrial fibrillation, gastroesophageal reflux disease (GERD), and hypertension. The only antithrombotic therapy that he was taking prior to the stroke was warfarin, but his International Normalized Ratio (INR) was in the therapeutic range only approximately 45% of the time based on a chart review. His recent CHA2DS2-VASc score is 5, and his HAS-BLED score is 2. Which of the following is the most appropriate medication for RD to use as secondary prophylaxis of cardioembolic stroke at this time? A. Dabigatran B. Clopidogrel C. Apixaban D. Warfarin

C. Apixaban is the most appropriate choice because RD is over the age of 75, had difficulty maintaining an INR in the therapeutic range (ideally at least 65-70% of time), and has a history of GERD.

GG is a 36-year-old female newly diagnosed with HIV infection. She is scheduled to start antiretroviral therapy and any recommended prophylaxis for opportunistic infections. She has never had an opportunistic infection. Her laboratory values are: CD4 count 50 cells/mm3, HIV RNA 125,000 copies/mL, BUN 14 mg/dL, SCr 0.6 mg/dL, Toxoplasma-specific IgG antibody positive, and glucose-6-phosphate dehydrogenase (G6PD) below the lower limit of the normal range. For which of the following organisms should GG receive prophylaxis at this time? A. Pneumocystis jirovecii and Mycobacterium avium complex B. Pneumocystis jirovecii, toxoplasmosis, and cytomegalovirus C. Pneumocystis jirovecii and toxoplasmosis D. Toxoplasmosis and Mycobacterium avium complex

C. Based on her CD4 count of 50 cells/mm3 and her seropositivity for toxoplasmosis, GG should receive primary prophylaxis for Pneumocystis jirovecii and toxoplasmosis. The threshold CD4 cell counts for prophylaxis are: <200 cells/mm3 for Pneumocystis jirovecii and <100 cells/mm3 for toxoplasmosis (with seropositivity). New guidelines do not strongly recommend prophylaxis for Mycobacterium avium complex with a CD4 count of 50cells/mm3 if effective antiretroviral therapy is begun.

Which of the following is the most appropriate baseline assessment to document in the chart to monitor the safety of donepezil in OB, an elderly patient recently diagnosed with Alzheimer's dementia? A. Mini-mental status exam (MMSE) score B. Thyroid-stimulating hormone (TSH) concentration C. Body weight D. Blood pressure

C. Clinically significant weight loss within the first few months of use of cholinesterase inhibitors has been demonstrated in patients with dementia. Therefore, body weight is the most appropriate baseline assessment to make in monitoring the safety of the cholinesterase inhibitor donepezil in OB.

If a patient suffered an extravasation of doxorubicin administered through their peripheral IV line, the most appropriate initial action after port removal would be: A. Active surveillance B. Application of a warm compress C. Dexrazoxane administration D. Sodium thiosulfate administration

C. Dexrazoxane is the best antidote for doxorubicin extravasation. It should be administered within 5 hours of the extravasation. There are efficacy data to support the use of dexrazoxane.

DP is a 55-year-old female with a history of heart failure with reduced ejection fraction (HFrEF), type 2 diabetes mellitus, and hypertension. She presents to clinic today for follow up after a recent hospitalization for heart failure (HF). DP is asymptomatic today, and her HF is classified in New York Heart Association (NYHA) class I. Her blood pressure is 110/78 mm Hg, heart rate is 68 bpm, height is 5'6", weight is 159 lb, and A1c is 7%. Her current medications include metformin 500 mg orally twice daily, carvedilol 25 mg orally twice daily, spironolactone 25 mg orally daily, and lisinopril 40 mg orally daily. Which of the following would be the most appropriate change to DP's treatment regimen to reduce her risk of hospitalization for HF? A. Increase carvedilol to 50 mg orally twice daily. B. Start ivabradine 5 mg orally twice daily. C. Start canagliflozin 100 mg orally daily. D. Start lixisenatide 10 mcg SC daily.

C. In the CANVAS trials of patients with type 2 diabetes and a high cardiovascular risk, patients receiving canagliflozin had a 33% reduction in heart failure hospitalizations compared with patients receiving placebo. Therefore, starting canagliflozin 100 mg orally daily is a good choice for DP who has type 2 diabetes and a high risk for cardiovascular events.

Which of the following initial antihypertensive combinations is most likely to be effective for lowering blood pressure in SD, a Black patient with hypertension (average systolic blood pressure 160 mm Hg; average diastolic blood pressure 96 mm Hg) based on the results of the CREOLE study and 2017 Guideline for Prevention, Detection, Evaluation, and Management for High Blood Pressure in Adults? A. Lisinopril plus hydrochlorothiazide B. Losartan plus chlorthalidone C. Amlodipine plus benazepril D. Amlodipine plus clonidine

C. In the CREOLE study, the combination of the calcium channel blocker amlodipine plus the angiotensin converting-enzyme (ACE) inhibitor perindopril or the thiazide-type diuretic hydrochlorothiazide was more effective for reducing blood pressure than perindopril plus hydrochlorothiazide in Black patients with uncontrolled hypertension. Some of these patients had been untreated (as is the case for SD), and others had been receiving antihypertensive monotherapy. Therefore, combination therapy with amlodipine plus the ACE inhibitor benazepril, or an amlodipine with a thiazide-type diuretic, is the best choice for controlling blood pressure in this Black patient with uncontrolled hypertension.

KA is a 6-year-old female who was recently diagnosed with mild persistent asthma and started using a budesonide dry powder inhaler (DPI) 90 mcg 1 puff twice daily and albuterol HFA 90 mcg/inhalation metered-dose inhaler (MDI) 2 puffs as needed for wheezing or shortness of breath. KA presents to clinic for follow-up management of her asthma. Her inhaler technique is evaluated, and KA demonstrates appropriate use. Although KA has been using the inhalers as prescribed, she has been awakening at night 1 or 2 times per week with shortness of breath that is relieved by her albuterol HFA MDI used with a spacer. Which of the following is the most appropriate adjustment in therapy at this time (the as-needed albuterol HFA inhaler would be continued)? A. Change the budesonide DPI to fluticasone HFA MDI 110 mcg 1 puff once daily. B. Add montelukast 10 mg by mouth daily at bedtime. C. Change the budesonide DPI to budesonide 80 mcg/formoterol 4.5 mcg HFA MDI 2 puffs twice daily. D. Increase the budesonide DPI dosage to 90 mcg 2 puffs twice daily.

C. KA's asthma is not well controlled based on her frequent use of albuterol for nighttime symptoms. Her current dosage of budesonide DPI is considered low-dose inhaled corticosteroid therapy in step 2 of chronic asthma treatment. She should be increased to step 3 chronic asthma treatment, with the preferred choice of increasing to a medium-dose inhaled corticosteroid or combination therapy with a low-dose inhaled corticosteroid + long-acting beta2-agonist in patients 5-11 years of age. Therefore, changing the budesonide DPI to budesonide 80 mcg/formoterol 4.5 mcg HFA MDI 2 puffs twice daily is the best choice for KA.

Previously conducted clinical trials have reported conflicting results regarding the role of fiber supplementation in irritable bowel syndrome (IBS). A meta-analysis was designed to determine whether fiber supplementation improves IBS symptoms. Which of the following is a potential disadvantage of the use of a meta-analysis? A. The effect size estimate is decreased B. The power to detect a significant difference is decreased C. The quality of the analysis is contingent on that of the studies included D. The heterogeneity of the studies is difficult to assess

C. Meta-analyses produce new data on effect estimates based directly on the clinical trials included in the analysis. Therefore, the inherent strengths and weaknesses of the included trials affect the findings of a meta-analysis.

BR is 66-year-old man with osteoarthritis and end-stage renal disease who presents today to the medication management clinic with a complaint of "I feel like my pain medications are running my life." He is currently receiving hydromorphone 4 mg orally every 4 hours and wakes up in the middle of the night to take a dose. When asked about the efficacy of this drug for him, he states, "It works well for my back pain, but it's difficult to keep up with so many doses." Which of the following would be the most appropriate alternative to hydromorphone for BR? A. Meperidine extended-release capsule 300 mg orally every 12 hours B. Morphine extended-release capsule 45 mg orally every 12 hours C. Methadone oral tablet 5 mg orally every 12 hours D. Oxycodone extended-release capsule 30 mg orally every 12 hours

C. Methadone is the safest choice for use in patients like BR with end-stage renal disease because its pharmacokinetics are not altered by renal failure, and methadone does not have any neurotoxic metabolites.

A 65-year-old Caucasian male with a history of hypertension was admitted 2-days ago with newly-diagnosed New York Heart Association (NYHA) class III acute decompensated heart failure. He has been successfully diuresed and started on oral drug therapy. His ejection fraction is 35%, BP is 165/94 mm Hg, heart rate is 93 bpm, and creatinine clearance (CrCl) is 35 mL/min. His current regimen includes lisinopril 30 mg orally daily, furosemide 20 mg orally daily, and metoprolol tartrate 25 mg orally twice daily. Which of the following management strategies is most appropriate for this patient at this time? A. Increase lisinopril from 30 mg orally daily to 40 mg orally daily B. Start spironolactone 25 mg orally daily C. Stop metoprolol tartrate 25 mg orally twice daily and start carvedilol 6.25 mg orally twice daily D. Stop lisinopril 30 mg orally daily and start sacubitril/valsartan 24 mg/26 mg orally twice daily

C. Metoprolol tartrate is not indicated for patients with heart failure based on the results of the COMET trial and subsequent guideline recommendations. The COMET trial demonstrated reduced mortality from the use of carvedilol instead of metoprolol tartrate in patients like this one with chronic heart failure, systolic dysfunction, and background treatment with diuretics and angiotensin converting-enzyme inhibitors. Therefore, switching from metoprolol tartrate to carvedilol is appropriate for this patient.

You are assisting a new orthopedic surgeon who is developing a set of standing orders for a particular procedure. The standing orders for postoperative pain medications include several different options for use after assessment of the patient's pain severity using a pain scale that will be included in the electronic medical record. Can the implementation of pain medication orders be at the discretion of the critical care nurses in the post-anesthesia care unit (PACU) based on patient pain assessment? A. No, because the physician must be physically present in the PACU to select the order. B. Yes, because this is within the scope of practice of critical care nurses in the PACU. C. No, because a licensed independent practitioner (LIP) must identify the order to use in this situation, and critical care nurses are not LIPs. D. Yes, because the order set will be approved by the pharmacy and therapeutics committee.

C. No, because a licensed independent practitioner (LIP) must identify the order to use in this situation, and critical care nurses are not LIPs.

Which of the following is considered a pitfall of publicly reporting outcomes in percutaneous coronary intervention (PCI)? A. Decreased public trust B. Interference with inter-hospital benchmarking C. Denial of care D. Compromised informed consent process

C. Public reporting and benchmarking can result in denial of PCI care to high-risk patients (who might stand to benefit despite their high risk) rather than treating all eligible patients because of fear that poor outcomes will compromise hospital benchmarking scores compared with other area hospitals where PCI is not performed.

The written asthma action plan that MR, an 11-year-old girl recovering from an acute asthma exacerbation, receives at the time of hospital discharge will be reviewed orally in accordance with recommendations from which of the following organizations? A. The National Quality Forum B. The Centers for Medicare & Medicaid Services C. The National Heart, Lung, and Blood Institute D. The National Committee for Quality Assurance

C. The National Heart, Lung, and Blood Institute (NHLBI) is the agency that drafts and periodically updates guidelines for the diagnosis and management of asthma (Expert Panel Report 3, or EPR-3), which provides specific examples and guidance regarding the importance of an asthma action plan as a tool to aid patients in self-management to prevent asthma-related hospitalizations and promote adherence to the treatment regimen. Oral review of MR's asthma action plan at the time of hospital discharge is consistent with recommendations from NHLBI.

You are a pharmacist who serves as a preceptor for a 4th year Doctor of Pharmacy student during an advanced pharmacy practice experience. One of the learning objectives for this experience is related to the AACP Core Entrustable Professional Activities for New Pharmacy Graduates in the Information Master domain. Specifically, by the end of the rotation the student is expected to be able to retrieve and analyze scientific literature to make a patient-specific recommendation. Based on the principles of adult learning, which of the following would be the best way for you to provide instruction and assess the achievement of this learning objective? A. Lead a weekly discussion session with the student on evidence-based medicine principles as they relate to the following topics: interpreting the primary literature and its application to practice; levels of evidence and evidence rating scales; best practices in clinical practice guideline development; and how to write a drug monograph, treatment guideline, and protocol. B. Provide weekly "predetermined" questions covering a wide range of clinically important issues, and in the last week of the experience, have the student complete an objective structured clinical exam (OSCE) with a different drug information question at each of four exam stations. C. Identify clinical questions that arise during patient care on a weekly basis, ask the student to identify appropriate sources of information to address the question, discuss what he/she has learned, and in the last week of the experience summarize the findings in writing and make specific recommendations about how the information should be applied to the patient case. D. Ask the student to spend time during the experience thinking about and preparing a report about evidence-based medicine, including a section about its history, major principles, use of data to make decisions, best practices for conducting systematic reviews and meta-analyses, and sources of error in clinical trials.

C. The stated objective is for the student to retrieve and analyze scientific literature to make a patient-specific recommendation. According to the principles of adult education, adults prefer to work autonomously, have prior learning and experiences, and see the relevance of what is being learned. Thus, learning activities that require practical application to an authentic task are likely to motivate learning. Moreover, to assess the student's achievement of this learning objective, asking the student to retrieve information and address a clinically relevant question related to a specific patient case is the only authentic way to determine whether the objective has been achieved. Of course, a student is likely to require guidance throughout the process, and the preceptor may need to provide some direct instruction as well as serve as a coach or facilitator, depending on the student's prior knowledge and experience performing this task.

Healthy Network has 26 community-based clinics located in 3 states and serves more than 2.5 million patients. Healthy Network employs dozens of clinical pharmacists who provide a variety of services, including anticoagulation therapy monitoring. Two patients in one clinic developed bladder cancer within 1 year after starting a direct oral anticoagulant (Drug A). Drug A was approved by the U.S. Food and Drug Administration (FDA) and European Medicines Agency 2 years ago and subsequently added to Healthy Network's formulary 18 months ago. The pharmacist wonders if the use of Drug A increases the risk of developing bladder cancer. There is no mention of bladder cancer in Drug A's FDA-approved prescribing information. The pharmacist wants to conduct an exhaustive search of the literature to answer this question. In addition to conducting a search of PubMed, which of the following sources would be the most useful in answering this question? A. FDA Adverse Event Reporting System (FAERS) B. Manufacturer's website C. Web of Science D. Scientific Electronic Library Online (SciELO)

C. Web of Science is the best choice because it is a comprehensive citation index that provides access to multiple databases and allows for in-depth exploration of specialized fields. Citations include links to similar items in journal articles, conference proceedings, and abstracts. Thus, a lot of scientific literature that is not indexed in PubMed can be searched. The FDA Adverse Event Reporting System (FAERS), under the Freedom of Information Act, allows citizen users to download and analyze adverse drug reaction (ADR) reports submitted to the agency. Although this information certainly would be useful from a research perspective, the data are "raw" and unanalyzed. Accessing these data might give the pharmacist a sense of the number of cases reported, but it would not provide information about the strength of evidence of the association or whether there is a causal relationship between Drug A use and bladder cancer. Given that the FDA-approved prescribing information does not include any information about an association between Drug A and the development of bladder cancer, it is unlikely that the manufacturer's website would voluntarily warn clinicians and patients about this serious adverse drug reaction (ADR) in the absence of published reports or a warning issued by the FDA. The manufacturer is required by law to report serious ADRs to the FDA. Therefore, an emerging safety issue reported to the manufacturer would also be reported to the FDA, and any official actions would be taken with FDA oversight. The Scientific Electronic Library Online (SciELO) is an electronic virtual library covering selected public health and social science journals published primarily in Latin America (Chile, Argentina, Brazil, Columbia, and Venezuela). Although searching this database might yield some information regarding the association between Drug A and bladder cancer if the medication is approved and widely used in Latin America, the limited scope of this database reduces the likelihood of uncovering any new information beyond that revealed through a PubMed search.

SM is a 3-year-old female patient with a recent history of asthma and seasonal allergies that currently are managed without prescription medications. She was in her usual state of health until 2 days ago when her mother reports hearing "whistling sounds" while breathing and a subjective fever this morning. The mother also states that SM has had a persistent nighttime cough for the last few weeks along with itchy eyes and a runny nose. SM was brought to the clinic because of respiratory distress and found to be tachypneic with a respiratory rate in the 60s and oxygen saturations of 89% to 91% on room air. She has received two 2.5-mg doses of albuterol via nebulizer and 1 mg/kg of intravenous methylprednisolone. SM was placed on oxygen via nasal cannula and is now showing signs of improvement in her respiratory symptoms. A respiratory viral panel was performed, and the results are negative. As the patient's condition stabilizes and the team prepares for transition back to home care, which of the following is the optimal therapeutic plan for this patient (with follow up with the primary care provider planned in 2 weeks)? A. Discharge home with albuterol HFA metered-dose inhaler with spacer and mask 90 mcg/inhalation 2 puffs every 6 hr as needed for wheezing or shortness of breath and loratadine 5 mg orally once daily B. Develop an asthma action plan and discharge home with albuterol 2.5 mg via nebulizer every 6 hr as needed for wheezing or shortness of breath and beclomethasone 42 mcg spray into each nostril twice daily C. Develop an asthma action plan and discharge home with albuterol HFA metered-dose inhaler with spacer and mask 90 mcg/inhalation 2 puffs every 6 hr as needed for wheezing or shortness of breath and mometasone furoate 50 mcg 1 spray into each nostril once daily D. Develop an asthma action plan and discharge home with albuterol 2.5 mg via nebulizer every 6 hr as needed for wheezing or shortness of breath, loratadine 5 mg orally once daily, and flunisolide 50 mcg spray into each nostril once daily

C. Whenever patients like SM with asthma present for health care, they should be provided with an asthma action plan for prevention and management of future asthma exacerbations to prevent hospitalizations. Furthermore, triggers should be addressed, which in this case seem to be an increase in allergic rhinitis symptoms for which she receives no prescription treatment. Intranasal corticosteroids are the preferred first-line therapy for seasonal allergic rhinitis in children because topical management of symptoms minimizes systemic exposure to corticosteroids and intranasal corticosteroids are more effective for managing symptoms than other therapies. The choice among intranasal corticosteroids for a patient 5 years of age or younger should be limited to one with a low systemic bioavailability, such as mometasone furoate (< 0.1%). Developing an asthma action plan with albuterol HFA metered-dose inhaler with spacer and mask 90 mcg/inhalation 2 puffs every 6 hr as needed for wheezing or shortness of breath and mometasone furoate 50 mcg 1 spray into each nostril once daily is the best choice for SM.

Which of the following was demonstrated in the Action to Control Cardiovascular Risk in Diabetes (ACCORD) trial comparing intensive therapy with standard therapy goals in patients with type 2 diabetes mellitus at high risk for cardiovascular (CV) events? A. Intensive therapy significantly reduced the incidence of CV events compared with standard therapy B. Intensive therapy significantly increased the incidence of CV events compared with standard therapy C. Intensive therapy did not significantly increase the incidence of serious adverse events compared with standard therapy D. Intensive therapy did not significantly reduce the incidence of CV events compared with standard therapy

D. In the ACCORD trial, intensive therapy with a systolic blood pressure (SBP) goal of less than 120 mm Hg did not significantly reduce the incidence of CV events compared with standard therapy with an SBP goal of less than 140 mm Hg in patients with type 2 diabetes at high risk for CV events. The annual rate of the primary outcome (a composite of nonfatal myocardial infarction, nonfatal stroke, or death from cardiovascular causes) was 1.87% in the intensive-therapy group and 2.09% in the standard-therapy group (p=0.20).

AB is a 65-year-old African-American woman admitted to the cardiac care unit (CCU) for New York Heart Association class IV acute decompensated heart failure. Physical examination reveals warm extremities and rales upon auscultation. Allergies: NKDA PMHx: non ST-segment elevation myocardial infarction with stenting (6 months ago), hypertension, osteoarthritis, bilateral renal artery stenosis Home MedicationsAmlodipine 5 mg orally dailyCarvedilol 25 mg orally twice dailyClopidogrel 75 mg orally dailyDiclofenac 75 mg orally twice dailyIsosorbide dinitrate 40 mg orally three times dailyPravastatin 20 mg orally daily Vital signs: BP 138/85 mm Hg, HR 92 bpm, RR 24/min, Ht 5'3", Wt 150 lb Pertinent Labs & Diagnostics: Na = 139 mEq/L K = 5.1 mEq/L Cl = 105 mEq/L HCO3 = 25 mEq/LBUN = 18 mg/dL SCr = 1.1 mg/dL Glucose = 84 mg/dLPCWP = 25 mm Hg, Cardiac index = 3 L/min/m2 Echocardiogram: left ventricular systolic function appears severely depressedVisually estimated ejection fraction = 20% Following her initial treatment, AB is stabilized and moved out of the CCU. Which of the following changes should be made to her current medication regimen to offer her the greatest mortality benefit? A. Increase amlodipine to 10 mg orally daily B. Increase carvedilol to 50 mg orally twice daily C. Initiate candesartan 4 mg orally daily D. Initiate hydralazine 25 mg orally three times daily

D. Initiating hydralazine 25 mg orally three times daily is correct because AB is already receiving isosorbide dinitrate, and the combination is recommended to reduce morbidity and mortality in patients intolerant to, or unable to receive, angiotensin converting-enzyme (ACE) inhibitors or angiotensin receptor blockers (ARBs). AB should not receive ACE Inhibitors or ARBs due to her underlying renal artery stenosis. Additionally, the combination of hydralazine and isosorbide dinitrate is recommended to reduce morbidity and mortality in self-identified African Americans receiving optimal therapy.

KS is a 3-year-old female who was initially started on amoxicillin 90 mg/kg/day by mouth for acute otitis media. She is brought back to clinic 4 days later with significant ear pain and fever. On examination, both tympanic membranes now have moderate bulging and are erythematous. KS' mother tells you that she was able to give her daughter only 2 days of medication due to a "bad rash" that covered KS' arms and legs. The best treatment plan for KS' acute otitis media at this time is to: A. Change amoxicillin to amoxicillin/clavulanate 45 mg/kg/day (amoxicillin component) by mouth for 7 days B. Change amoxicillin to ceftriaxone 50 mg/kg/day IM for 5 days C. Change amoxicillin to azithromycin 10 mg/kg/day by mouth for 5 days D. Change amoxicillin to cefuroxime 30 mg/kg/day by mouth every 12 hr for 7 days

D. KS has not truly failed to respond to initial treatment with amoxicillin, but she did have a hypersensitivity reaction to amoxicillin. A third-generation cephalosporin such as cefuroximeis recommended for patients with acute otitis media and a penicillin allergy.

Which of the following statements most accurately reflects the impact of continuing to smoke cigarettes after myocardial infarction (MI)? A. It negatively impacts only angina-related quality of life B. It does not impact health-related quality of life C. It positively impacts only angina-related quality of life D. It negatively impacts health-related quality of life

D. Patients who continue to smoke cigarettes after MI experience worse overall health-related quality of life and more angina compared with patients who stop smoking after MI.

AP-MOM study, a case-control study, assessed whether exposure to acetaminophen in utero is associated with childhood obesity. The study relied on mothers reporting their past acetaminophen use before, during, and after their most recent pregnancy. Prior to administering the telephonic survey, the investigators shared with the participants their hypothesis that childhood obesity may be linked to maternal acetaminophen use. Cases were obese children who were between 3 and 5 years old. Controls were matched by age, sex, and county. Which of the following types of bias is the APAP-MOM study most prone to? A. Classification bias B. Selection bias C. Publication bias D. Recall bias

D. Recall bias occurs when information about an exposure or an outcome is inaccurately recalled or remembered. The APAP-MOM study relied on participant reporting of past acetaminophen use and was initially based on participant subjective recall of that information, which may be difficult to remember. Given that the investigators shared with the participants their hypothesis regarding the link between maternal acetaminophen use and childhood obesity, the mothers of obese children probably were more likely to report instances of acetaminophen use during pregnancy (which may or may not have actually occurred). Classification bias (which occurs when an exposure or outcome or both is categorized incorrectly), selection bias (which occurs when the method for choosing study participants results in exposures that are different from those that would have occurred without the bias), and publication bias (which influences the decision to publish study findings) are unlikely sources of bias in the APAP-MOM study.

A 48-year-old HIV-positive man has the following fasting lipid panel: total cholesterol 240 mg/dL, LDL-C 160 mg/dL, HDL-C 25 mg/dL, and triglycerides 220 mg/dL. His antiretroviral regimen is darunavir + ritonavir + emtricitabine + tenofovir disoproxil fumarate. His family history is significant for cardiovascular disease. His father died of a heart attack at 48 years of age. Which of the following drug therapies for dyslipidemia should he be started on at his next clinic visit, if any? A. Sustained-release niacin B. Pravastatin + fenofibrate C. Antidyslipidemic therapy is not needed in this patient D. Rosuvastatin

D. Rosuvastatin is the best choice for this patient, especially given his family history (his father died of a heart attack at age 48). A significant drug interaction would not be expected with his antiretroviral regimen.

BK has been taking an HIV regimen consisting of rilpivirine + tenofovir alafenamide + emtricitabine for 6 months. His viral load is 400 copies/mL and his CD4 count is 200 cells/mm3. His regimen is switched to lamivudine + abacavir (co-formulated) and darunavir + ritonavir. After a month on this regimen, he reports fatigue, nausea, and vomiting and he has noticed a rash. The most likely cause for these symptoms is: A. Immune reconstitution inflammatory syndrome (IRIS) B. A drug interaction between abacavir and ritonavir C. Metabolic syndrome from protease inhibitor therapy D. Hypersensitivity to abacavir

D. The nature and timing of BK's symptoms are typical of a hypersensitivity reaction to abacavir. Most clinics use HLA-B* 5701 genotypic testing to determine whether patients are at risk for abacavir hypersensitivity syndrome before starting the drug.

A new director of nursing (DON) has started working at your facility and she has been asked to follow up on the facility's Star rating. She has questions for you about how the Star rating is calculated and is having difficulty finding information. Which of the following organizational websites do you direct your DON to? A. The RAND Corporation B. American Medical Directors Association C. American Geriatrics Society D. Centers for Medicare and Medicaid Service

D. This information is located on the Centers for Medicare and Medicaid Services website. It includes information about how the score is calculated, the specific Star ratings for specific nursing homes by zip code, and an explanation for how the information is collected.

PB is a 60-year-old woman with hypothyroidism. Her levothyroxine has been titrated to a stable dosage and her thyroid-stimulating hormone (TSH) level has normalized since her last visit. How soon should she have her next TSH level measured? A. 6 months from now B. 1 month from now C. 2 months from now D. 3 months from now

A. 6 months is correct. After the TSH levels normalize and a stable dosage is established, levels may be drawn after 6 months and then annually.

A pharmacist made a common dispensing error involving a specific drug and dosage form. The patient was harmed, and the pharmacist is upset about the error. At a recent pharmacy staff meeting, information was provided about the prevalence of this error in your hospital. The organization's philosophy is to support a "just culture" for patient safety. Which of the following was involved in this scenario using this philosophy? A. Human error B. At-risk behavior C. Reckless conduct D. Punitive sanctions

A. This scenario involved human error. The error was not intentional. In a just culture, the pharmacist who made the error would be consoled and supported.

Which of the following is a nonpharmacologic therapy that should be recommended to all patients with severe chronic obstructive pulmonary disease (COPD) based on guidelines from the Global Initiative for Chronic Obstructive Lung Disease (GOLD)? A. Environmental assessment B. Pulmonary rehabilitation C. Psychotherapy D. Annual spirometry

B. According to the GOLD guidelines, pulmonary rehabilitation is an essential nonpharmacologic therapy recommended for all individuals with severe COPD.

Patients receiving medication therapy management within a health system are surveyed to assess their satisfaction with the pharmacist-provided services. Individual responses to the survey are rated on a scale from 1 to 5, with 1 being highly dissatisfied and 5 being highly satisfied. Which of the following would be the best method to determine the central tendency of these data? A. Mean B. Mode C. Median D. Standard deviation

C. This patient satisfaction survey involves a LIKERT scale, which are ordinal data. The median is the middle value of rank-ordered data points, which is an appropriate measure of central tendency for ordinal data or interval/ratio data that are not normally distributed due to the presence of outliers. The mean and mode do not accurately reflect central tendency of this type of data. Standard deviation is a measure of variation or data dispersion, not a measure of central tendency.

Which of the following is the minimum frequency recommended by the American Diabetes Association (ADA) for scanning intermittently scanned continuous glucose monitoring (CGM) in patients receiving multiple injections of insulin daily? A. Every 8 hours B. Every 12 hours C. Every 24 hours D. When signs or symptoms of hypoglycemia are present

A. The ADA recommends testing scanning intermittently scanned CGM values at least every 8 hours in patients receiving multiple insulin injections.

The Hospital Conditions of Participation are created by which of the following organizations? A. Centers for Medicare & Medicaid Services (CMS) B. American Hospital Association C. The Joint Commission (TJC) D. State boards of health

A. CMS develops the Hospital Conditions of Participation. Hospitals must comply with these requirements to receive federal funding.

Which of the following is the primary responsibility of an institutional review board (IRB)? A. Protect the rights and welfare of study participants B. Evaluate the need for informed consent in a study proposal C. Review a study protocol for scientific rigor D. Monitor study participants for study drug-related adverse events

A. An IRB can perform all these tasks (e.g., evaluating the need for informed consent in a study proposal, reviewing a study protocol for scientific rigor, and monitoring study participants for study drug-related adverse events), but the primary responsibility is to protect the rights and welfare of human research subjects.

KP is 33-year-old woman who gave birth to a baby boy 2 hours ago. The infant weighed 4.4 kg at birth. During her pregnancy, KP was treated with ceftriaxone after she was diagnosed with gonococcal cervicitis. After birth, the baby is noted to have red, inflamed eyes that contain purulent exudate. A swab of the exudate is sent to the lab for evaluation, and empiric therapy is ordered. What is the optimal therapy for this neonate? A. Ceftriaxone IV and erythromycin ophthalmic ointment B. Ciprofloxacin ophthalmic solution C. Cefotaxime IV D. Erythromycin ophthalmic ointment

A. Intravenous ceftriaxone should be given to this neonate based on the timing of his clinical presentation and the mother's history of gonococcal disease. Cefotaxime may be used as an alternative to ceftriaxone because of concerns about biliary sludge formation in children treated with the latter. Erythromycin ophthalmic ointment also should be used because of the possibility of exposure to chlamydia during delivery.

A 19-year-old woman experiences burning and itching on urination and notices an uncharacteristic foul smell to her urine. She tells her physician that she can't come to the clinic because she is leaving for a trip in 2 days but she is sure she has a urinary tract infection (UTI) because she has had infections "just like this" in the past (2 years and 4 years ago). She reports that she has not recently been sexually active. Her health record contains a complicated history of allergies that include penicillins, sulfonamides, and fluoroquinolones. She cannot remember exactly what the reactions were but notes that some manifested as nausea, others as a rash, and one as tongue swelling. The physician would like to provide a prescription for a course of oral antibiotic therapy. Which of the following would be the best empiric choice for her infection? A. Nitrofurantoin 100 mg every 12 hr x 5 days B. Amoxicillin 500 mg every 8 hr x 7 days C. Cephalexin 500 mg every 12 hr x 7 days D. Doxycycline 100 mg every 12 hr x 7 days

A. Nitrofurantoin is the best choice for this patient. There is strong evidence of success with a 5-day duration of treatment for UTIs.

A 1-year-old girl is brought to her pediatrician by her father because of a suspected "ear infection." The father explains that his daughter has been tugging at her right ear for the last 2 days and has been extremely irritable. About a week and a half ago, she completed a course of amoxicillin for an ear infection on the same side with a ruptured eardrum. The patient experienced prompt symptom resolution following administration of the antibiotic. The father has not seen any discharge coming from his daughter's ear as was the case 3 weeks ago, but the symptoms are similar. On physical examination of the right ear, the pediatrician finds significant erythema of the middle ear and a swollen tympanic membrane with healing of the site of perforation of the eardrum. The patient weighs 18 kg and is 38 inches tall. Which of the following is the most appropriate approach to treating this patient? A. Amoxicillin-clavulanate 750 mg (amoxicillin component) orally every 12 hr x 10 days B. Amoxicillin 800 mg orally every 12 hr x 10 days C. Cefdinir 260 mg orally every 12 hr x 7 days D. Ceftriaxone 900 mg IM every 24 hr x 3 days

A. Oral amoxicillin-clavulanate is the most appropriate therapy for this patient's acute otitis media (AOM). She received amoxicillin for AOM within the past 30 days, so it would not be prudent for her to receive this drug alone again. The dosage of amoxicillin-clavulanate is important because this 18-kg patient should receive 80-90 mg/kg/day of the amoxicillin component. A dosage of 750 mg (amoxicillin component) every 12 hours provides approximately 83 mg/kg/day. Finally, because she is only 1 year of age, the duration of therapy should be 10 days.

A 72-year-old man presents to the emergency department complaining of abdominal pain, cramping, and profuse, watery diarrhea that slowly worsened over the past week. He completed a course of ciprofloxacin for a urinary tract infection (UTI) 3 weeks ago. The patient is admitted to the hospital, and a stool sample is sent to the lab. Results of a stool Clostridioides difficile (C. difficile) toxin assay and glutamate dehydrogenase (GDH) test are positive. He is given fluids and started on oral vancomycin. The patient is discharged after 4 days and completes a 10-day course of vancomycin therapy with full resolution of his initial gastrointestinal signs and symptoms. Two weeks after completing vancomycin therapy he again experiences a sudden onset of profuse, watery diarrhea. On return to the clinic, his stool C. difficile toxin assay again is positive. Other reported lab test results include a white blood cell (WBC) count of 14.1 x 103/mm3 and serum creatinine (SCr) of 1.2 mg/dL. Which of the following is the most appropriate management for this patient's ongoing diarrhea based on the 2017 clinical practice guidelines for C. difficile infection (CDI) from the Infectious Diseases Society of America (IDSA) and Society for Healthcare Epidemiology of America (SHEA)? A. Fidaxomicin 200 mg orally twice daily x 10 days B. Vancomycin 125 mg orally four times daily x 10 days C. Metronidazole 500 mg orally three times daily and probiotics each day for 14 days D. Vancomycin 500 mg orally four times daily and probiotics each day for 14 days

A. The patient is experiencing his first recurrence of C. difficile infection (CDI) following his initial infection 3 weeks ago. According to the 2017 IDSA/SHEA clinical practice guidelines for CDI, in cases where vancomycin was used to treat the initial episode, a course of oral fidaxomicin for 10 days should be used to treat a first recurrence.

A 22-year-old man with no known drug allergies has been hospitalized on the general medicine floor for the past 3 days with a severe, purulent skin and soft tissue infection of his right upper extremity. He was initially started on daptomycin 800 mg IV daily for treatment of the infection. His past medical history is significant for depression (for which he takes sertraline) and injection drug use (testing positive for heroin use). His serum creatinine is 0.9 mg/L, and the rest of his laboratory values are within normal limits. Today, cultures from initial incision and drainage (I&D) return positive for Staphylococcus aureus with the following susceptibilities: Cefazolin- R Clindamycin- R Erythromycin- R Gentamicin- S Levofloxacin- I Rifampin- S Linezolid- S Tetracycline- S TMP-SMX- S Vancomycin- S His condition has improved substantially since he was first hospitalized and he is due to be discharged on hospital day 4 to complete 6 more days of antibiotic therapy. Which of the following is the most appropriate oral antibiotic therapy for him to use to complete therapy after discharge? A. Trimethoprim-sulfamethoxazole B. Daptomycin C. Rifampin D. Linezolid

A. The patient's initial severe purulent SSTI was managed by daptomycin and I&D. Culture results from the initial I&D of the infected site returned positive for methicillin-resistant Staphylococcus aureus (the pathogen is resistant to the beta-lactam antibiotic cefazolin) with susceptibilities that can be used to guide therapeutic decision making. Trimethoprim-sulfamethoxazole would be the most appropriate choice for this patient because of his substantial clinical improvement since admission, the susceptibility of the isolated pathogen to the drug, and the need for oral therapy due to his history of injection drug use.

PS is a 77-year-old woman with a 6-month history of excessive worrying, which started after her husband passed away suddenly. She currently takes allopurinol 100 mg orally once daily for a history of recurrent gout attacks, pravastatin 80 mg orally at bedtime for dyslipidemia, and lorazepam 0.25 mg orally four times daily as needed for anxiety. All of these medications were prescribed by her primary care provider, and PS notes that she takes a dose of lorazepam "once or twice a week." The psychiatrist who you work with diagnoses her with generalized anxiety disorder (GAD) today. In addition to continuing lorazepam, which of the following is the best drug therapy for PS to start today for her GAD? A. Duloxetine 30 mg orally daily B. Buspirone 5 mg orally three times daily C. Quetiapine 25 mg orally at bedtime D. Hydroxyzine 50 mg orally at bedtime

A. The use of a selective serotonin reuptake inhibitor (SSRI) or serotonin norepinephrine reuptake inhibitor (SNRI) is appropriate as first-line therapy for the long-term treatment of GAD, and the SRNI duloxetine is approved by the Food and Drug Administration for the treatment of GAD. The continued use of lorazepam in this patient is considered appropriate until long-term maintenance therapy (i.e., duloxetine) for GAD has a chance to take effect.

Mrs. Johnson who was admitted from the community for an elective hip replacement develops postoperative complications, including pneumonia on day 5. She was admitted with community-acquired pneumonia 2 months ago and treated successfully with intravenous (IV) ceftriaxone and azithromycin. The patient's pulmonary status deteriorates today. She requires mechanical ventilation for respiratory support, becomes hemodynamically unstable, and the use of vasopressor support is imminent. Mrs. Johnson has normal renal function. You are asked to recommend initial empiric antibiotic therapy for the patient. At your institution, piperacillin-tazobactam is the treatment of choice for hospital-acquired pneumonia. Your antibiogram demonstrates 83% susceptibility to the drug among respiratory Gram-negative pathogens and a 42% rate of methicillin-resistant Staphylococcus aureus (MRSA). Which of the following antibiotics would provide the narrowest spectrum of antimicrobial activity that is considered appropriate for empiric use in Mrs. Johnson? A. Piperacillin-tazobactam plus gentamicin plus vancomycin B. Piperacillin-tazobactam alone C. Piperacillin-tazobactam plus gentamicin D. Piperacillin-tazobactam plus vancomycin

A. The use of vancomycin, which provides MRSA coverage, in combination with dual antibiotic therapy that provides Gram-negative coverage (e.g., piperacillin/tazobactam and gentamicin) is the best choice for Mrs. Johnson. She is at increased risk for MRSA due to her recent IV antibiotic therapy and the high rate of MRSA (greater than 10%) at your institution, and she is also at risk for drug-resistant Gram-negative pathogens.

In which of the following patient populations is a reduction in cardiovascular (CV) morbidity and mortality most likely to occur from the use of icosapent ethyl based on the results of the REDUCE-IT study? A. Patients with a history of myocardial infarction (MI) B. Patients with heart failure C. Patients with chronic kidney disease (CKD) D. Patients with hypertension

A. This answer is correct. The REDUCE-IT study was a randomized, placebo-controlled study of the impact of icosapent ethyl, a highly purified form of eicosapentaenoic acid ethyl ester, on first major atherosclerotic events (CV death, nonfatal MI, nonfatal stroke, coronary revascularization, or unstable angina). Patients with established CV (e.g., history of MI) or with diabetes plus other CV risk factors were enrolled. Compared with placebo, the incidence of a first major atherosclerotic event was significantly reduced in patients receiving icosapent ethyl.

Mrs. Smith is a 93-year-old patient with chronic obstructive pulmonary disease who is admitted to your facility with influenza (lab confirmed). She becomes hemodynamically unstable and is transferred to the intensive care unit where she is intubated for failure to oxygenate. She is treated for 4 days with a neuraminidase inhibitor. On day 4, her vasopressor requirements begin to decrease, and a ventilator weaning trial is initiated. On day 5, her oxygen requirements increase and pressure support for the ventilator is resumed. She is restarted on maximum doses of norepinephrine. A bronchoscopic alveolar lavage culture reveals the presence of Gram-negative bacilli 75,000 CFU/mL, and she is clinically diagnosed with pneumonia. Which of the following types of pneumonia does Mrs. Smith have? A. Ventilator-associated pneumonia (VAP) B. Community-acquired pneumonia (CAP) C. Hospital-acquired pneumonia (HAP) D. Healthcare-associated pneumonia (HCAP)

A. VAP is correct because this pneumonia developed more than 48 to 72 hours after endotracheal intubation.

LK is a patient with active breast cancer who was transitioned to warfarin after 6 months of enoxaparin therapy. LK comes to the clinic 1 week after beginning her fourth cycle of a regimen of carboplatin and gemcitabine for her metastatic breast cancer. Her international normalized ratio (INR) today is 4. She reports no significant bruising or bleeding. She experiences nausea and vomiting for three days following each cycle of chemotherapy, which she treats with lorazepam and ondansetron. Which of the following is the most likely cause of her INR elevation? A. Initiation of ondansetron for nausea management B. A drug-drug interaction with chemotherapy C. Initiation of lorazepam for nausea treatment D. Three days of nausea and vomiting after chemotherapy

D. Experiencing 3 days of chemotherapy-induced nausea and vomiting could influence the INR in multiple ways. The patient's dietary intake of vitamin K may have been reduced due to nausea and vomiting, resulting in an elevated INR.

For which of the following disease states have Healthcare Effectiveness Data and Information Set (HEDIS) measures been established that can be used to compare the performance of individual institutions based on national benchmarks? A. Dyslipidemia and chronic kidney disease B. Heart failure and peripheral artery disease C. Obesity and stroke D. Hypertension and diabetes

D. HEDIS measures have been established for medication management for asthma, persistence of beta-blocker treatment after a heart attack, controlling high blood pressure, comprehensive diabetes care, breast cancer screening, antidepressant medication management, childhood immunization status, and adult body mass index assessment. The controlling high blood pressure and comprehensive diabetes care HEDIS measures can be used for comparison of performance in caring for patients with hypertension or diabetes.

You join a new service at your health system and meet a patient who is being effectively treated for hospital-acquired pneumonia (HAP) caused by Pseudomonas aeruginosa. The organism was recovered via bronchoalveolar lavage and is susceptible to the ceftazidime that has been used for treatment. The patient has completed 5 days of therapy. The physician asks if treatment can be discontinued. The most appropriate duration of therapy for this patient is: A. 5 days B. 7 days C. 14 days D. Until radiographic results have returned to normal

B. A 7-day course of antibiotics is warranted in patients like this one who have HAP and have responded to initial therapy based on the results of the study by Chastre et al and guidelines from the Infectious Diseases Society of America and the American Thoracic Society.

Which of the following patient populations with type 2 diabetes is a less stringent A1c goal (e.g., 7.5-8.0%) based on current recommendations from the American Diabetes Association (ADA) Standards of Medical Care in Diabetes? A. Patients with newly diagnosed diabetes B. Patients with heart failure with reduced ejection fraction and a history of myocardial infarction C. Patients with a life expectancy exceeding 5 years D. Patients with a strong support system

B. A less stringent A1c goal is appropriate for patients with type 2 diabetes, heart failure with reduced ejection fraction, and a history of myocardial infarction. The current ADA Standards of Medical Care for Diabetes recommends that A1c goals be individualized. Factors that should be taken into consideration for a less stringent A1c goal for a person with type 2 diabetes include: history of severe hypoglycemia, limited life expectancy, long-standing diabetes (e.g., 10 years), established vascular complications, extensive comorbid conditions, limited resources/support system, and/or difficulty attaining goals despite maximal combination therapy and counseling.

Which of the following is a potential confounding variable in the study of e-cigarettes vs. nicotine replacement for smoking cessation? A. The reliance on self-reports of abstinence from smoking starting 2 weeks following the quit date. B. The relatively long study duration compared with the 6 months used for most smoking cessation studies. C. The use of a self-selected alternative smoking cessation treatment (i.e., not necessarily the assigned treatment) after the first 30 days of the 1-year study. D. The favorable influence of symptomatic improvement (i.e., reductions in cough and phlegm production) on adherence to the assigned treatment.

C. Given that patients were permitted to use alternative smoking cessation aids other than the assigned treatment after the initial 30 days, the 1-year abstinence rates in each group might be attributable to treatment other than the assigned one. These factors, whether known or unknown, are considered confounders because they are not randomly (or equally) assigned to participants during the study.

According to the 2012 Infectious Diseases Society of America (IDSA) clinical practice guideline for acute bacterial rhinosinusitis (ABRS) in children and adults, which of the following patients is likely to have ABRS? A. A 41-year-old man with HIV infection who presents with a 5-day history of nasal congestion, facial pain, and cough B. A 62-year-old woman who presents with a 2-week history of facial pressure, anosmia, and nasal congestion C. A 27-year-old man who presents with a 2-day history of cough and a fever of 101oF D. A 49-year-old woman with chronic obstructive pulmonary disease (COPD) who was discharged from the hospital 4 days ago and now presents with shortness of breath, headache, and purulent nasal discharge for the past 24 hours

B. According to the 2012 IDSA clinical practice guideline, diagnosis with bacterial rhinosinusitis requires meeting one of the following criteria: persistent symptoms of rhinosinusitis for >10 days without improvement OR onset of severe symptoms including fever >102°F, purulent nasal discharge, or facial pain lasting 3-4 days beyond the beginning of illness OR worsening of symptoms following initial onset and improvement after 5-6 days that include fever, headache, or increase in nasal discharge. This patient has been experiencing rhinosinusitis symptoms of facial pressure, anosmia, and nasal congestion for 14 days. Therefore, the likelihood of ABRS is high in this patient.

A 33-year-old, HIV-infected woman just finished treatment for an outbreak of genital herpes simplex virus (HSV). It was her first occurrence, and her lesions and symptoms resolved after 15 days. Her HIV viral load is undetectable at <48 copies/mL, and her CD4 cell count is 558 cells/mm3. She is currently receiving an antiretroviral drug regimen formulated as a single pill taken once a day to treat her HIV infection and finds this regimen to be much easier than her previous regimen consisting of four different pills per day. She tells her primary care physician that she does not want to have another herpes outbreak and asks about continuing treatment for HSV. Which of the following would be the best option for the physician to recommend to the patient for HSV suppression? A. Famciclovir B. Acyclovir C. Ganciclovir D. Valacyclovir

B. Acyclovir, famciclovir, and valacyclovir are all reasonable options for HSV suppressive therapy. All three drugs require similar dosing regimens (i.e., at least twice daily administration) and offer similar efficacy for HSV suppression. Acyclovir is the best choice because it has the lowest cost.

LG is a 34-year-old African-American HIV-positive patient who develops hemolytic anemia while taking trimethoprim-sulfamethoxazole for Pneumocystis jirovecii prophylaxis. Which of the following alternative agents should be used in its place? A. Dapsone 100 mg orally once daily B. Atovaquone 1.5 g orally once daily C. Azithromycin 1.2 g orally once weekly D. Levofloxacin 500 mg orally once daily

B. Atovaquone 1.5 g orally once daily is one of the best alternatives to trimethoprim-sulfamethoxazole for Pneumocystis pneumonia prophylaxis.

Which of the following represents the best pharmacotherapy plan for a patient with history of stroke and depression taking fluoxetine 40 mg daily who now requires dual antiplatelet therapy with aspirin 81 mg daily and clopidogrel 75 mg daily after intracoronary stent placement? A. Change clopidogrel to prasugrel B. Change fluoxetine to citalopram C. Decrease fluoxetine to 20 mg daily D. Increase clopidogrel to 150 mg daily

B. Changing fluoxetine to citalopram is the best choice for this patient because fluoxetine use is not recommended in combination with clopidogrel due to CYP 2C19 inhibition by fluoxetine (clopidogrel is a prodrug activation of which involves CYP 2C19). Changing to citalopram avoids this interaction because citalopram is not an inhibitor of CYP 2C19.

A patient receiving empiric treatment for hospital-acquired pneumonia (HAP) experiences a hives-like reaction to piperacillin-tazobactam on the first day of therapy. Which of the following antibiotics would be considered appropriate intravenous therapy to replace the piperacillin-tazobactam therapy and avoid a cross-hypersensitivity reaction (without any additional allergy testing)? A. Dalbavancin B. Ciprofloxacin C. Ceftazidime/avibactam D. Ceftaroline

B. Ciprofloxacin is the best choice for this patient because it is not known to have any direct cross reactivity with beta-lactam antibiotics. Aztreonam would also be an acceptable choice because it has very low cross reactivity in patients with type I mediated allergic reactions to penicillins. Finally, utilization of a carbapenem (including desensitization) may be necessary in some situations where resistance to agents, such as ciprofloxacin and aztreonam, is suspected.

Two days ago a male patient was empirically initiated on intravenous (IV) levofloxacin, gentamicin, and vancomycin therapy for hospital-acquired pneumonia that resulted in hemodynamic instability. Today, the results of a culture of a deep sputum sample obtained 2 days ago demonstrate the presence of an Escherichia coli (E. coli) isolate that is susceptible to levofloxacin. The patient is now hemodynamically stable, although he requires 2 L/min of oxygen to maintain saturations greater than 92%. The patient meets the criteria for your IV-to-oral medication switch program. Which of the following represents the best course of action for this patient at this time? A. Continue IV levofloxacin, gentamicin, and vancomycin to complete 7 days of antibiotic therapy B. Discontinue gentamicin and vancomycin and change IV levofloxacin to oral levofloxacin to complete 7 days of antibiotic therapy C. Discontinue vancomycin and continue IV levofloxacin and gentamicin to complete 7 days of antibiotic therapy D. Discontinue levofloxacin, gentamicin, and vancomycin and initiate oral penicillin to complete 7 days of antibiotic therapy

B. Discontinuing gentamicin and vancomycin and continuing therapy with an antibiotic that is effective for treating pneumonia and active against the isolated pathogen (i.e., levofloxacin) is the most appropriate course of action in this patient. Oral levofloxacin is indicated for the treatment of pneumonia because its oral bioavailability is high and the drug penetrates into the respiratory epithelial lining fluid (i.e., the site of action for treatment of extracellular bacterial pneumonia). This patient meets the criteria for switching from IV to oral therapy and this choice is supported by the Infectious Diseases Society of America and American Thoracic Society for managing hospital-acquired and ventilator-associated pneumonia.

A 66-year-old woman is in the hospital for an elective hip replacement. Two days after surgery, she spikes a fever and becomes hypotensive, with a blood pressure of 95/56 mm Hg. She is given fluids, but remains hypotensive and is transferred to the intensive care unit (ICU). Vasopressors and broad-spectrum antibiotics are initiated. She is also mechanically ventilated to provide respiratory support and has a nasogastric tube inserted. Four days later, she develops profuse, watery diarrhea, which has now ceased. Her white blood cell (WBC) count had been hovering at 20 x 103/mm3 but today it increased to 40 x 103 /mm3, and her serum lactate rose to 5 mmol/L. A CT scan of her abdomen reveals significant inflammation throughout the colon and acute ileus. Results of polymerase chain reaction testing for Clostridioides difficile (C. difficile) are positive. Which of the following options is the best management for this situation? A. Vancomycin 125 mg orally every 6 hr B. Vancomycin 500 mg orally every 6 hr, metronidazole 500 mg IV every 8 hr, and rectal vancomycin 500 mg every 12 hr C. Vancomycin 125 mg orally every 6 hr and metronidazole 500 mg IV every 8 hr D. Vancomycin 500 mg orally every 6 hr, intravenous immunoglobulin 400 mg/kg daily, and rectal vancomycin 500 mg every 12 hr

B. This patient has a C. difficile infection (CDI) based on his symptoms (profuse diarrhea that has stopped, elevated WBC count, colonic inflammation, and ileus) as well as PCR testing that reveals C. difficle. This patient's episode would be classified as initial, fulminant based on the diagnosis of CDI and presence of ileus. Recommendations for treating initial episode, fulminant CDI include vancomycin 500 mg orally every 6 hours. Metronidazole 500 mg IV every 8 hours and rectal vancomycin 500 mg every 12 hours should also be used because ileus is present and there is concern that vancomycin administered orally may not reach the site of infection in adequate concentrations.

A 59-year-old man presents to your clinic complaining of chest pain when he takes a breath, cough, and increased sputum production over the past 3 days. He states the sputum is very thick and brown in color and that his cough is "worse than normal." His past medical history is significant for chronic obstructive pulmonary disease (COPD), cigarette smoking (1 pack per day), dyslipidemia, and hypertension. Chest radiography performed at the clinic reveals the presence of a large infiltrate in the right lower lobe. His vital signs are within normal limits except for his temperature, which is elevated at 101.2oF. His lab test results are within normal limits except for his white blood cell (WBC) count, which is slightly elevated. He is diagnosed with community-acquired pneumonia (CAP). Which of the following is the most appropriate management for this patient? A. Outpatient treatment, azithromycin 500 mg orally x 1, followed by 250 mg orally daily x 4 days B. Outpatient treatment, levofloxacin 750 mg orally daily x 10 days C. Inpatient treatment, ceftriaxone 2 g IV daily + azithromycin 500 mg IV daily x 7 days D. Inpatient treatment, piperacillin-tazobactam 4.5 g IV every 6 hr + levofloxacin 750 mg IV daily x 10 days

B. This patient has been diagnosed with CAP as an outpatient and has no signs of clinical instability other than a temperature > 100oF. Therefore, he qualifies for outpatient, oral therapy. He should receive either (1) a respiratory fluoroquinolone alone or (2) a beta-lactam antibiotic plus a macrolide antibiotic, all given orally, because of his COPD. Levofloxacin at a dose of 750 mg orally daily for 10 days is the best choice for this patient

You are a pharmacist who has been charged with enhancing the quality of life in patients served by the HIV clinic at your health system. Which of the following strategies do you recommend using because it has been shown to enhance quality of life measures in patients with HIV? A. Provide free condoms to all clinic patients B. Switch as many patients to protease inhibitor-based regimens as possible C. Simplify the antiretroviral regimen when feasible and in accordance with guidelines D. Increase the frequency of clinic appointments for all patients

C. A few small studies have shown that simplification of the antiretroviral drug regimen can improve quality of life scores in patients with HIV.

In a cohort study, the risk of a hospital emergency department visit for uveitis in the 90 days after initiation of DRUG A in men at least 60 years old was compared with a matched cohort of men who did not take DRUG A. The risk of uveitis (the primary outcome) was 0.61% in the cohort of 1000 men exposed to DRUG A and 0.41% in a group of 1000 men unexposed to DRUG A, odds ratio (OR) = 1.5 (95% CI 1.01 to 2.28). Which of the following is the best interpretation of the results from this study? A. The absolute increase in risk of uveitis among men exposed to DRUG A compared with unexposed men is not consistent with the odds ratio. B. There is a 150% higher risk of uveitis among men exposed to DRUG A compared with men who were not exposed to DRUG A. C. The difference in the risk of uveitis among men exposed to DRUG A when compared with men who were not exposed to DRUG A is statistically significant. D. It is not possible to determine whether the difference in the risk of uveitis among men exposed and not exposed to DRUG A is statistically significant because a p-value is not reported.

C. A measures of association confidence interval that does not touch or cross the number "1" (i.e., the line of no effect) is indicative of a statistically significant difference. In this trial, the 95% confidence interval 1.01 to 2.28 did not touch or cross 1, and it corresponds to a p-value less than 0.05. Therefore, the difference is statistically significant.

JM is a male medical student who recently learned that he is HIV positive. He knows you as the pharmacist at the hospital and asks you what resources he can turn to for the best evidence-based consensus data supporting his antiretroviral therapy (ART) and the need for any opportunistic infection prophylaxis. Which of the following is the best resource to recommend for JM? A. Ryan White HIV/AIDS Program B. AIDS Drug Assistance Program (ADAP) C. AIDSinfo.nih.gov website D. The Medical Letter

C. The www.AIDSinfo.nih.gov website is the best choice for JM because it contains evidence-based national guidelines on ART and management of opportunistic infections. Because this patient is a medical student with a high level of health care literacy, he will be able to comprehend the information on this website. The guidelines are periodically updated and tend to provide more up-to-date information than The Medical Letter.

Mrs. Johnson, a female with hospital-acquired pneumonia and septic shock that developed 5 days after elective hip replacement surgery, received the maximum recommended amount of normal saline for treatment of her shock. The use of vasopressor support is imminent. Which of the following is the most appropriate next step in the treatment of her shock based on Surviving Sepsis Campaign international guidelines for management of sepsis and septic shock? A. Dextrose 5% in water (D5W) B. Hydroxyethyl starch C. Albumin D. Hydrocortisone

C. Albumin is a guideline-recommended choice for the management of sepsis and septic shock, albeit with a weak recommendation and low quality evidence. Nevertheless, albumin is the best choice for Mrs. Johnson.

Mrs. Johnson is a female with hospital-acquired pneumonia and septic shock that developed 5 days after elective hip replacement surgery. You are asked to recommend a target blood glucose concentration while she is in septic shock. She does not have any history of diabetes mellitus, and an A1c has not been measured. Mrs. Johnson's average blood glucose value over the past 3 days is 245 mg/dL. Which of the following is the guideline-supported upper limit for blood glucose that should be targeted in Mrs. Johnson? A. 110 mg/dL B. 140 mg/dL C. 180 mg/dL D. Blood glucose need not be corrected in patients without a history of diabetes mellitus

C. In the randomized NICE-SUGAR study comparing intensive blood glucose control (81-108 mg/dL) with a less intensive blood glucose target (less than 180 mg/dL) in critically ill patients (Finfer et al), there was no significant difference in 90-day mortality but the incidence of severe hypoglycemia was significantly higher with intensive blood glucose control than less intensive control. Therefore, an upper limit for blood glucose of 180 mg/dL is the best choice for Mrs. Johnson.

A patient with hospital-acquired pneumonia (HAP) is becoming hemodynamically unstable. His current blood pressure, 80/50 mm Hg, is well below his normal hypertensive state of 140/95 mm Hg. He weighs 95 kg. Which of the following is the intravenous therapy of first choice to raise his blood pressure and regain hemodynamic stability? A. 1000 mL of normal saline (0.9% sodium chloride) B. 2000 mL of albumin (20 g, 20%) C. 3000 mL of Lactated Ringer's D. Vasopressin 0.03 units/min

C. Lactated Ringer's 30 mL/kg (i.e., roughly 3000 mL for this 95-kg patient) is recommended in the Surviving Sepsis Campaign international guidelines for managing severe sepsis and septic shock for initial fluid resuscitation in patients with severe sepsis or septic shock. Emerging data suggest that "balanced" solutions, such as Lactated Ringer's, may prevent nephrotoxicity better than normal saline.

A 74-year-old male presents to your clinic with a large abscess on the anterior aspect of his left hip. The abscess is fluctuant and painful. His most recent lab test results from a prior clinic visit were notable for a serum creatinine of 1.1 mg/L. The patient weighs 90 kg. The clinic physician has determined that it is safe to drain his moderately purulent abscess in the clinic today. An initial Gram stain reveals Gram positive cocci in clusters. In addition to appropriate incision and drainage, which of the following therapeutic options would be the most appropriate empiric treatment for this patient at this time? A. Cefazolin 2 g IV every 8 hr B. Linezolid 600 mg orally every 12 hr C. Trimethoprim/sulfamethoxazole 2 double-strength tablets orally every 12 hr D. Dalbavancin 1500 mg IV as a one-time dose

C. Oral trimethoprim/sulfamethoxazole is appropriate for empiric treatment of this patient's moderately purulent skin and soft tissue infection in combination with incision and drainage. Narrow-spectrum agents, such as cefazolin, should be reserved for definitive therapy of purulent infections. Broad-spectrum agents, such as linezolid, should be reserved for severe purulent infections. Newer parenteral antibiotics, such as oritavancin and dalbavancin, probably should be reserved for severe purulent infections.

DF is a 56-year-old female who presents to the emergency department with profound left lower leg swelling, erythema, tenderness, and extreme pain in response to light touch, although no purulence is noted. Her past medical history is significant for congestive heart failure, diabetes mellitus, hypertension, and hyperlipidemia. DF is diagnosed with lower extremity cellulitis due to an acute bacterial skin and soft tissue infection (SSTI). Which of the following is the most appropriate empiric treatment for DF? A. Oral trimethoprim/sulfamethoxazole and cephalexin B. IV vancomycin C. IV vancomycin and piperacillin-tazobactam D. Oral cephalexin

C. The patient's presentation is consistent with an acute necrotizing SSTI, so initial broad-spectrum empiric treatment with piperacillin-tazobactam and vancomycin in addition to emergent surgical debridement is required. Oral antibiotics are not acceptable for this patient's severe SSTI. Vancomycin alone is recommended as empiric therapy for purulent SSTIs but it should be combined with piperacillin-tazobactam for this patient's possible necrotizing nonpurulent SSTI.

Which of the following patients is a candidate for treatment with a proprotein convertase subtilisin/kexin type 9 (PCSK9) inhibitor based on the 2018 American College of Cardiology (ACC)/American Heart Association (AHA) guideline on the management of blood cholesterol to reduce atherosclerotic cardiovascular risk in adults? A. A patient with known clinical atherosclerotic cardiovascular disease (ASCVD) on maximally tolerated statin therapy with a 50% reduction in LDL-C and no previous use of lipid-lowering therapies other than statins B. A patient with a baseline LDL-C of 160 mg/dL and no known clinical atherosclerotic cardiovascular disease (ASCVD) who is intolerant to one statin C. A patient with known clinical atherosclerotic cardiovascular disease (ASCVD) on maximally tolerated statin therapy plus ezetimibe with a less than 50% reduction in LDL-C D. A patient with heart failure on maximally tolerated statin therapy with an LDL-C of 130 mg/dL

C. This is correct. The 2018 ACC/AHA guideline recommends the use of PCSK9 inhibitors for secondary prevention of cardiovascular events in patients on maximally tolerated statin therapy with clinical ASCVD who do not achieve a 50% reduction in LDL-C or who have an LDL-C greater than 70 mg/dL. Ezetimibe is preferred over PCSK9 inhibitors in this patient population, with a PCSK9 inhibitor added as a third agent (with a statin plus ezetimibe).

Which of the following oral therapies may provide relief for a patient who develops peripheral neuropathy after receiving paclitaxel? A. Acetaminophen 650 mg every 6 hours B. Celecoxib 200 mg every 12 hours C. Gabapentin 300 mg every 8 hours D. Duloxetine 60 mg daily

D. According to current American Society of Clinical Oncology guidelines, duloxetine is preferred as first-line treatment for chemotherapy-induced peripheral neuropathy.

LL, a 12-year-old female, presents with her mother to the pediatrician complaining of a rapid-onset sore throat, a slight cough, and hoarseness in her voice. Her mother states that when she has measured LL's temperature this morning it was 102.2oF. LL has a history of seasonal allergies in the fall, and she takes loratadine only during that season. In a review of systems, she complains of isolated throat pain, without any rhinorrhea, sinus pressure, or headache. She has documented allergies to amoxicillin (rash) and sulfa (swelling). The pediatrician diagnoses pharyngitis, but wants to confirm a diagnosis of group A streptococcal (GAS) pharyngitis. Which of the following is the most appropriate approach for confirming this diagnosis? A. Obtain a throat culture for GAS B. Her symptoms alone confirm the diagnosis of GAS pharyngitis, so no further testing is required C. Perform a urine antigen test to detect the presence of GAS D. Obtain a throat swab for a rapid antigen detection test (RADT) for GAS

D. Any patient who presents with symptoms suggesting acute pharyngitis that are not consistent with a viral origin should be tested for GAS using a RADT. This patient's age of 12 years, rapid-onset sore throat, and fever are consistent with GAS pharyngitis, but a diagnosis cannot be made on based clinical symptoms alone and requires the use of a RADT. This testing is more than 95% sensitive for GAS, and results take 24-48 hours less time than a throat culture. Therefore, obtaining a throat swab for a RADT for GAS is the best choice to confirm a diagnosis of GAS pharyngitis in LL.

You are the consultant pharmacist in a nursing home, and you are reviewing the medication regimen for an 87-year-old male patient. According to the American Geriatrics Society 2019 Beers criteria for potentially inappropriate medication use in older adults, which of the following medications on the patient's profile should you make a recommendation to switch to an agent with a better risk/benefit profile? A. Insulin B. Teriparatide C. Warfarin D. Doxazosin

D. Doxazosin is included in the Beers criteria because it can cause orthostatic hypotension and the drug has an unfavorable risk-versus-benefit profile when used in the older adult population.

Vancomycin is chosen as directed therapy for a patient with culture-proven, methicillin-resistant Staphylococcus aureus (MRSA) pneumonia. Which of the following represents a guideline-recommended dosing strategy for this patient? A. Achieving peak plasma vancomycin concentrations of at least 50 mg/L 1 hour after a dose B. Employing a dosage regimen that allows the free drug concentration to exceed the minimum inhibitory concentration (MIC) for at least 40% of the dosing interval C. Maintaining trough plasma vancomycin concentrations of 15-20 mg/L D. Maintaining an area under the concentration curve (AUC) of 400-600 mg/L*24 hr

D. Maintaining an AUC of 400-600 mg/L*24 hr is recommended for treating serious MRSA infections in the newest vancomycin guidelines.

A female with type 2 diabetes mellitus and acute kidney injury was admitted to your facility's general medical ward because of diabetic ketoacidosis 4 days ago. The patient's blood glucose concentrations have stabilized but her pulse oximetry is now 89%. The patient has rales and crackles in the right lower lobe, and a chest X-ray confirms the presence of pneumonia. Sputum samples have been obtained and are being processed by the laboratory. The patient has not received any antibiotics within the past 5 months, and she is hemodynamically stable. The prevalence of methicillin-resistant Staphylococcus aureus (MRSA) at your hospital is 8%, and piperacillin-tazobactam covers 92% of local Gram-negative respiratory pathogens. Which of the following antibiotics would provide the narrowest antimicrobial spectrum of activity that is considered appropriate to use empirically in this patient? A. Piperacillin-tazobactam plus gentamicin B. Piperacillin-tazobactam plus vancomycin C. Piperacillin-tazobactam plus gentamicin plus vancomycin D. Piperacillin-tazobactam alone

D. Piperacillin-tazobactam alone is the best choice for this patient because it provides the narrowest antimicrobial spectrum of activity therapy that is guideline-supported. Notably, it will provide coverage for most Gram-negative respiratory pathogens at this institution as well as methicillin-susceptible Staphylococcus aureus. The addition of gentamicin is not needed because resistance among local Gram-negative respiratory pathogens to piperacillin-tazobactam is less than 10%. The addition of vancomycin, which provides MRSA coverage, is not needed because the patient has not received antibiotics within the past 90 days and the local hospital prevalence of MRSA is less than 20%. Furthermore, the patient is hemodynamically stable (i.e., not in septic shock) and she is not mechanically ventilated, so she is not at increased risk for MRSA. Finally, the addition of other agents such as gentamicin and vancomycin could further complicate this patient's acute kidney injury without providing additional benefit.

A 78-year-old man presents to the emergency department with chief complaints of shortness of breath, cough, and "his heart feeling like it was going to beat out of his chest." His cough started 2 days ago and has gotten progressively worse. He awoke this morning feeling weak and feverish, so he checked his temperature, which was 100.9oF. His past medical history is significant for atrial fibrillation (managed by amiodarone), hypertension (managed by lisinopril and hydrochlorothiazide), and myocardial infarction 4 years ago. His current vital signs are: blood pressure 158/93 mm Hg, heart rate 121 beats per minute, respiratory rate 25 breaths per minute, temperature 101.5oF, and oxygen saturation 92% on room air. He is diagnosed with community-acquired pneumonia and admitted to the general medicine floor. Which of the following is the most appropriate empiric antibiotic therapy for this patient? A. Cefuroxime + levofloxacin B. Moxifloxacin C. Ertapenem + linezolid + azithromycin D. Ceftriaxone + doxycycline

D. The combination of an antipneumococcal beta-lactam antibiotic plus a macrolide antibiotic or a respiratory fluoroquinolone alone is appropriate for treating CAP in patients hospitalized in a non-intensive care unit. Empiric coverage for Streptococcus pneumoniae, Haemophilus influenzae, and atypical pathogens is needed in patients with suspected CAP. This patient has a history of atrial fibrillation for which he takes amiodarone and presents with a rapid, racing heart, so drugs that can prolong the QTc interval (e.g., fluoroquinolones, macrolide antibiotics) should be avoided. Therefore, the use of theanti-pneumococcal beta-lactam antibiotic ceftriaxone plus the tetracycline doxycycline is appropriate to provide the necessary antimicrobial coverage without prolonging the QTc interval in this patient.

A 26-year-old man presents to the emergency department (ED) with repeated fevers, a rash on his legs, and complaints of bilateral knee pain. On examination, the physician notices significant swelling and nodules on both knees. He orders an aspirate from both knees, and samples are sent to the microbiology lab. Initial inspection of the samples shows a white blood cell (WBC) count >50,000 cells/mm3. While awaiting culture results, a presumptive diagnosis of septic arthritis is made. The microbiology lab reports the Gram stain as gram-negative diplococci, and follow-up nucleic acid amplification tests confirm the presence of Neisseria gonorrhoeae (N. gonorrhoeae). Which of the following is the most appropriate therapy for this patient? A. Ciprofloxacin 500 mg orally twice daily B. Cefixime 400 mg orally daily C. Ceftriaxone 250 mg IM x 1 dose plus azithromycin 1 g orally x1 dose D. Ceftriaxone 1 g IV daily plus doxycycline 100 mg orally twice daily for 7 days

D. The recommended therapy for N. gonorrhoeae is ceftriaxone 1 g IV daily until symptoms have resolved plus doxycycline 100 mg orally twice daily for 7 days if chlamydial infection has not been excluded.

MS, an 8-year-old female, presents with her parents to the outpatient pharmacy to refill her fluticasone 115 mcg/salmeterol 21 mcg HFA metered-dose inhaler (MDI), which she uses as 1 puff twice daily. Her parents recently read about bone health problems from steroid use in children with asthma and are concerned about the long-term effects of the medication on MS, especially since her grandmother was recently diagnosed with osteoporosis. Which of the following counseling points regarding MS's inhaled corticosteroid therapy should be shared with the patient and her parents at this time? A. It provides a high, yet effective dose of inhaled corticosteroid to control her asthma and minimizes her risk for adverse effects on bone health only if taken with a multivitamin or calcium in conjunction with vitamin D as part of her overall plan for health maintenance. B. It provides an effective but high dose of inhaled corticosteroid to control her asthma but it increases her risk for adverse effects on bone health unless the dose is decreased and taken with a multivitamin or calcium in conjunction with vitamin D as part of her overall plan for health maintenance. C. It provides the lowest effective dose of inhaled corticosteroid to control her asthma, but to avoid adverse effects on bone health she should use the inhaler only as needed for wheezing or shortness of breath. D. It provides the lowest effective dose of inhaled corticosteroid to control her asthma and minimize her risk for adverse effects on bone health. She might consider taking a multivitamin or calcium in conjunction with vitamin D as part of her overall plan for health maintenance.

D. Studies show that although inhaled corticosteroids can suppress linear growth and increase the risk for osteopenia and osteoporosis in children with asthma, use of the lowest effective dose is the best way to minimize these risks. The use of a multivitamin or calcium and vitamin D supplementation might be considered to optimize bone health, although studies have not definitively shown that calcium and vitamin D supplementation reduce the risk of adverse effects on bone health from inhaled corticosteroid use in children with asthma.

MJ is a 50-year-old African-American female who presents to clinic after a recent hospitalization for heart failure (HF). She has a past medical history of asthma, hypertension, HF, and acute kidney injury that developed during her recent hospitalization. Today she has no visible edema but complains of dyspnea on exertion. Her blood pressure is 143/82 mm Hg, heart rate is 79 bpm, creatinine clearance (CrCl) is 40 mL/min, serum potassium is 4.5 mEq/L, and ejection fraction is 32%. Her current medication regimen includes albuterol HFA 90 mcg 2 puffs inhaled every 6 hours as needed for bronchospasm, lisinopril 20 mg orally daily, bisoprolol 10 mg orally daily, furosemide 20 mg orally twice daily, and spironolactone 25 mg orally daily, which was started immediately before discharge from the hospital 5 days ago. Which of the following therapy modifications would offer the greatest reduction in her risk of rehospitalization for HF? A. Increase lisinopril to 40 mg orally daily. B. Stop lisinopril and after an appropriate washout period start sacubitril/valsartan 49 mg/51 mg orally twice daily. C. Increase spironolactone to 50 mg orally daily. D. Stop bisoprolol and after an appropriate washout period start carvedilol 25 mg orally twice daily.

Sacubitril/valsartan was proven superior to enalapril for reducing the risks of death and hospitalization for HF in the PARADIGM-HF trial (McMurray et al.). Therefore, stopping lisinopril for 36 hours and starting sacubitril/valsartan 49 mg/51 mg twice daily represents the best choice to reduce the risk of future hospitalizations for HF in this patient whose blood pressure is elevated despite receiving the target angiotensin converting-enzyme inhibitor dosage.


Related study sets

Sem 3 - Unit 6 - Cognition (Itellectual Disabilities; Chromos Abn; Fetal Alcohol Syndrome)

View Set

D271 Brazil Final (info after midterm)

View Set

Crash Course: Race & Prejudice - Ch 12

View Set

Exam 2 David Myers, Psychology in Modules 11th

View Set

Object-Oriented Programming (OOP) concept

View Set

Econ Exam - Chapter's Assignment Review

View Set